Advanced Pharm: Antibiotics & Antiinfectives

Pataasin ang iyong marka sa homework at exams ngayon gamit ang Quizwiz!

16. The provider can prescribe a monoclonal antibody medication for the patient with which condition? a. Crohn disease b. Acne c. Psoriatic arthritis d. Multiple myeloma

ANS: C Monoclonal antibodies (mAbs) are prescribed for psoriatic arthritis. Proteasome inhibitors are prescribed for multiple myeloma. This classification of medications is not appropriate for either Crohn's or acne.

8. An adolescent has recently developed scattered comedones across the forehead & nose. Which medication does the provider recommend to the patient & parents? a. Benzoyl peroxide b. Topical clindamycin c. Topical erythromycin d. Topical retinoids

ANS: A Benzoyl peroxide is a first-line drug for mild to moderate acne. Other topical antibiotics & retinoids are used when first-line therapy fails.

41. Before prescribing methenamine, it is important for the provider to review the patient's history for evidence of which problem? a. Elevated blood urea nitrogen and creatinine b. History of reactions to antibiotic agents c. Possibility of pregnancy d. Previous resistance to antiseptic agents

ANS: A Methenamine should not be given to patients with renal impairment, because crystalluria can occur. There is no cross-reactivity between methenamine and antibiotic agents. Methenamine is safe for use during pregnancy. There is no organism drug resistance to methenamine.

8. A patient receiving doxazosin presents for a routine evaluation. Which assessment finding related to the medication therapy would be most concerning to the provider? a. Blood pressure 96/58 b. Dizziness when standing c. Increased nasal congestion d. Diminished ejaculate volume

ANS: A Hypotension, dizziness, and nasal congestion are all adverse effects of doxazosin, an α1-adrenergic antagonist. However, hypotension would be the most concerning because it can lead to inadequate peripheral tissue perfusion. Diminished ejaculate volume is seen with administration of 5-α-reductase inhibitors, not with doxazosin administration.

18. A patient received 500 mg of azithromycin at 0800 as a first dose. What order will the provider write for the amount and time of the second dose of azithromycin? a. 250 mg at 2000 the same day b. 500 mg at 2000 the same day c. 250 mg at 0800 the next day d. 500 mg at 0800 the next day

ANS: C Azithromycin generally is given as 500 mg on the first day and then 250 mg/day for the next 4 days, so the second dose would be 24 hours after the first dose.

14. Which cephalosporin may the prescriber order to treat meningitis? a. Cefaclor b. Cefazolin c. Cefoxitin d. Cefotaxime

ANS: D Cefotaxime has increased ability to reach the cerebrospinal fluid (CSF) and to treat meningitis. Cefaclor, cefazolin, and cefoxitin do not reach effective concentrations in the CSF.

9. A patient has a skin infection and the culture reveals methicillin-resistant Staphylococcus aureus (MRSA). What medication treatment should the provider prescribe for this patient? a. Cefaclor b. Cefazolin c. Cefotaxime d. Ceftaroline

ANS: D Ceftaroline is a fifth-generation cephalosporin with a spectrum similar to third-generation cephalosporins but also with activity against MRSA. Cefaclor is a second-generation cephalosporin. Cefazolin is a first-generation cephalosporin. Cefotaxime is a third-generation cephalosporin.

22. A provider is teaching a patient who has taken glucocorticoids for over a year about glucocorticoid withdrawal. Which statement by the patient indicates a need for further teaching? a. "I should reduce the dose by half each day until I stop taking the drug." b. "I will need to have cortisol levels monitored during the withdrawal process." c. "The withdrawal schedule may take several months." d. "If I have surgery, I may need to take the drug for a while, even after I have stopped."

ANS: A Glucocorticoid therapy can suppress adrenal function, so withdrawal should be done slowly to allow recovery of adrenal function. Reducing the dose of a glucocorticoid by half each day is not recommended. Patients should have their cortisol levels monitored to determine when therapy can be stopped. The withdrawal schedule may take several months. Patients who have stopped the drug may still experience adrenal insufficiency in times of physiologic stress, such as surgery.

14. An adolescent is brought to the emergency department after consuming a bottle of extended-release acetaminophen tablets between 8 and 10 hours ago. Which intervention will the provider order? a. Acetylcysteine administration b. Activated charcoal administration c. Hemodialysis d. Gastric lavage

ANS: A Acetylcysteine is the specific antidote for acetaminophen overdose. It is 100% effective when given within 8 to 10 hours after ingestion and may still have some benefit after this interval. Activated charcoal and gastric lavage are effective only if given before the medication is absorbed. Hemodialysis is not indicated.

30. A patient with a urinary tract infection is given a prescription for TMP/SMZ. When reviewing the drug with the patient, the provider learns that the patient has type 1 diabetes mellitus and consumes alcohol heavily. What action will the provider take? a. Prescribe a different antibiotic for this patient. b. Order daily blood glucose determinations while giving TMP/SMZ. c. Suggest that the patient take a potassium supplement while taking TMP/SMZ. d. Suggest that the patient avoid excessive fluid intake while taking TMP/SMZ.

ANS: A Alcoholics are likely to be folate deficient and have an increased risk of megaloblastic anemia when taking TMP/SMZ, so withholding this drug in this population is recommended. TMP/SMZ shares hypersensitivity reactions with oral sulfonylurea-type hypoglycemics that are used with type 2 diabetes mellitus, so it is not necessary to assess the blood glucose level more often. TMP/SMZ can cause hyperkalemia, so potassium supplements are contraindicated. Patients taking TMP/SMZ should consume more fluids to maintain renal blood flow and prevent renal damage.

73. A male patient is being treated for benign prostatic hyperplasia & has stopped taking his alpha-adrenergic antagonist medication because of ejaculatory difficulties. Which medication does the nurse expect the provider to prescribe? a. Alfuzosin [Uroxatral] b. Prazosin [Minipress] c. Silodosin [Rapaflo] d. Tamsulosin [Flomax]

ANS: A Alfuzosin is used for BPH & does not interfere with ejaculation. All of the other drugs have ejaculatory side effects. Prazosin may be useful for BPH, but it is not approved for this use.

78. A young female patient is seen in a rural clinic after complaining of abdominal pain. The patient is wearing dirty clothing & is barefoot. The provider orders a complete blood count, which shows that the patient is anemic. The provider suspects that this patient has which of the following infestations? a. Ancylostomiasis (hookworm) b. Ascariasis (giant roundworm) c. Enterobiasis (pinworm) d. Trichuriasis (whipworm)

ANS: A Ancylostomiasis is most common when hygiene is poor & the patient habitually goes barefoot. Symptomatic anemia may occur in menstruating women or undernourished individuals. Ascariasis is usually asymptomatic. Enterobiasis is characterized by perianal itching. Trichuriasis is usually asymptomatic but may cause rectal prolapse if the worm burden is very large.

12. A college track star with a history of seasonal allergies is diagnosed with bipolar disorder. After the prescriber teaches the patient about newly ordered lithium, which statement by the patient indicates the need for further teaching? a. "I can continue to use ibuprofen as needed for muscle pain." b. "I need to drink extra fluids before & during exercise." c. "I should stop taking antihistamines while taking lithium." d. "I should report muscle weakness & tremors to my provider."

ANS: A Because ibuprofen, a nonsteroidal anti-inflammatory drug (NSAID), can increase lithium levels as much as 60%, it should not be used by patients taking lithium. Aspirin does not have this effect. Lithium induces polyuria in 50% to 70% of patients, so patients should be advised to drink extra fluids, especially during exercise. Antihistamines have anticholinergic effects, which cause urinary hesitancy; this can be uncomfortable when patients experience the polyuria associated with lithium use. Muscle weakness & tremors can occur with lithium; tremors can be treated with β blockers or by altering the lithium regimen.

20. A patient taking a glucocorticoid for arthritis reports feeling bloated and peripheral edema is noted. Which action by the provider is relevant to the care of this patient? a. Asking the patient about sodium intake b. Obtaining a blood glucose level c. Suggesting the patient limit potassium intake d. Discontinuing the drug

ANS: A Because of their mineralocorticoid activity, glucocorticoids can cause sodium and water retention and potassium loss. Asking about the sodium intake can help the nurse evaluate this patient. Although glucocorticoids can affect glucose tolerance, this patient does not have signs of hyperglycemia. Patients with sodium and water retention should be encouraged to increase their potassium intake. Telling a patient to stop taking the drug is incorrect, because this side effect can be managed and this action does not take into consideration the benefits versus the risks.

42. A patient is taking sertraline [Zoloft] for depression, and the provider orders azithromycin [Zithromax] to treat an infection. What will the nurse do? a. Contact the provider to discuss an alternative to azithromycin. b. Request an order for a different antidepressant medication. c. Request an order to reduce the dose of sertraline. d. Withhold the sertraline while giving the azithromycin.

ANS: A Both sertraline & azithromycin prolong the QT interval, & when taken together, they increase the risk of fatal dysrhythmias. Because the antibiotic is used for a short time, it is correct to consider using a different antibiotic. Reducing the dose of sertraline does not alter the combined effects of two drugs that lengthen the QT interval. Sertraline should not be stopped abruptly, so withholding it during antibiotic therapy is not indicated.

10. A patient who has been taking sertraline for depression was prescribed azithromycin to treat an infection by a provider at an after-hours clinic. What action will the primary care provider take to address the risk this combination of medication has posed for the patient? a. Discontinue the azithromycin and write an order for an alternative antibiotic. b. Discontinue the sertraline and write an order for a different antidepressant medication. c. Reduce the sertraline dosage while taking azithromycin. d. Withhold the sertraline until the azithromycin therapy is completed.

ANS: A Both sertraline and azithromycin prolong the QT interval, and when taken together, they increase the risk of fatal dysrhythmias. Because the antibiotic is used for a short time and because the patient was already taking sertraline, it is correct to consider using a different antibiotic. Reducing the dose of sertraline does not alter the combined effects of two drugs that lengthen the QT interval. Sertraline should not be stopped abruptly, so withholding it during antibiotic therapy is not indicated. Additionally, it is important to reinforce the need to tell all providers that sertraline is being taken.

11. A patient is to undergo orthopedic surgery, and the prescriber will order a cephalosporin to be given preoperatively as prophylaxis against infection. Which generation of cephalosporin will the provider order? a. First b. Second c. Third d. Fourth

ANS: A First-generation cephalosporins are widely used for prophylaxis against infection in surgical patients, because they are effective, less expensive, and have a narrower antimicrobial spectrum than second-, third-, and fourth-generation cephalosporins.

65. A patient diagnosed with HIV and mucocutaneous HSV is being treated with foscarnet after failing treatment with acyclovir. After 2 weeks, the patient's dose is increased to 90 mg/kg over 2 hours from 40 mg/kg over 1 hour. The patient reports numbness in the extremities and perioral tingling. What action will the provider take? a. Order a serum calcium level. b. Treat for a potential foscarnet overdose. c. Order a creatinine clearance level. d. Order IV saline to be given before the next dose.

ANS: A Foscarnet frequently causes hypocalcemia and other electrolyte and mineral imbalances. Paresthesias, numbness in the extremities, and perioral tingling can indicate hypocalcemia, so a calcium level should be drawn. These are not signs of foscarnet overdose. Nephrotoxicity may occur, but these are not signs of renal complications, so a creatinine clearance is not indicated. If nephrotoxicity occurs, prehydration with IV saline is indicated to reduce the risk of renal injury.

74. A patient complains of painful urination. A physical examination reveals vesicles on her labia, vagina, & the foreskin of her clitoris. Which medication will the provider prescribe? a. Acyclovir b. Azithromycin c. Metronidazole d. Tinidazole

ANS: A Genital herpes can be treated with acyclovir, famciclovir, or valacyclovir, which are antiviral medications. Azithromycin, metronidazole, & tinidazole are antibiotics & do not have antiviral effects.

3. A patient is given a new prescription for potassium penicillin G given intravenously (IV) every 8 hours and gentamicin given IV every 12 hours. Which is the best schedule for administering these drugs? a. Give the penicillin at 0800, 1600, and 2400; give the gentamicin [Garamycin] at 1800 and 0600. b. Give the penicillin at 0800, 1600, and 2400; give the gentamicin [Garamycin] at 1200 and 2400. c. Give the penicillin at 0600, 1400, and 2200; give the gentamicin [Garamycin] at 0600 and 1800. d. Give the penicillin every 8 hours; give the gentamicin [Garamycin] simultaneously with two of the penicillin doses.

ANS: A Gentamicin should never be administered concurrently with penicillin, because they will interact, and the penicillin may inactivate the aminoglycoside. All the other options show concurrent administration.

23. A patient who takes a glucocorticoid reports having tarry stools but denies gastric pain. After ruling out anemia and determining that the patient is not in danger, which action will the provider take? a. Prescribing an antiulcer medication. b. Counseling the patient to use over-the-counter antacids. c. Reassuring the patient not to worry unless there is gastric pain. d. Discontinuing the glucocorticoid immediately.

ANS: A Glucocorticoid therapy can increase the risk of gastric ulcer and possibly GI bleeding. Treatment with antiulcer medications is indicated, but not with OTC antacids. Gastric pain is usually decreased because of the glucocorticoids, so absence of gastric pain is not reassuring. The glucocorticoid should be withdrawn slowly, not immediately.

2. The provider is assessing a newly diagnosed patient for short-term complications of diabetes. What evaluation does this assessment include? a. Serum blood sugar results for hyperglycemia b. Cranial nerve testing for peripheral neuropathy c. Pedal pulse palpation for arterial insufficiency d. Auscultation of the carotids for bruits associated with atherosclerosis

ANS: A High blood sugar, low blood sugar, and ketoacidosis are short-term complications of diabetes. Microvascular and macrovascular complications, such as peripheral neuropathy, are long-term complications of diabetes. Arterial insufficiency and atherosclerosis also are long-term complications of diabetes.

19. A patient taking high doses of a glucocorticoid develops weakness in the muscles of the upper arms and in the legs. What action will the provider take? a. Reducing the dose of the glucocorticoid medication b. Encourage the patient to restrict sodium intake. c. Reassure the patient that this is an expected side effect. d. Discontinue the medication immediately

ANS: A High-dose glucocorticoid therapy can cause myopathy, manifesting as weakness. If muscle weakness occurs, the dose should be reduced. Reducing the sodium intake is recommended to minimize sodium and water retention, not to decrease muscle weakness. Muscle weakness is not an expected side effect, because it indicates myopathy. It is incorrect to tell the patient to stop taking the drug, because a glucocorticoid must be withdrawn slowly to allow time for recovery of adrenal function.

4. A 2-month-old infant is scheduled to receive the first dose of DTaP. What information will the provider include in instructions given to the parents? a. "Usually reactions are mild; a low-grade fever is most common." b. "Most children do not experience any reaction." c. "Seizures are common and may require anticonvulsant medication." d. "The most common reaction is a rash that develops into itchy blisters."

ANS: A Mild reactions to the first dose of the DTaP vaccine are common and most often are manifested by a low-grade fever, fretfulness, drowsiness, and local reactions of swelling and redness. At least 50% of children experience reactions. Seizures are not common. Itchy vesicles do not appear with the DTaP vaccine.

6. A provider assesses an adolescent male patient who has been receiving androgen therapy for hypogonadism via a transdermal patch. The patient's last clinic visit was 4 weeks earlier. Which part of the interval history is of greatest concern to the provider? a. Five-pound weight gain b. Increased growth of pubic hair c. Rash at the site of the patch d. Presence of acne

ANS: A Patients receiving testosterone may experience edema secondary to sodium and water retention. Treatment involves discontinuing the drug and giving diuretics if needed. Growth of pubic hair is an expected effect. A rash at the site of transdermal application is a common effect. Acne is an expected effect.

13. A patient with bipolar disorder who has been taking lithium for several years has developed a goiter. When the serum tests reveal hypothyroidism what will the provider prescribe for this patient? a. Levothyroxine therapy b. Increasing the lithium dose c. Iodine supplements daily d. Propylthiouracil

ANS: A Patients taking lithium may experience reduced incorporation of iodine into the thyroid hormone, resulting in goiter and hypothyroidism. Administration of levothyroxine or withdrawing the lithium will reverse both. Increasing the lithium dose or prescribing propylthiouracil will make this worse. Iodine supplements are not indicated.

44. A patient with HIV who takes protease inhibitors develops tuberculosis and will begin treatment. Which drug regimen will the provider prescribe for this patient? a. Isoniazid, pyrazinamide, ethambutol + rifabutin b. Isoniazid, pyrazinamide, ethambutol c. Isoniazid, rifampin, pyrazinamide, ethambutol d. Isoniazid + rifabutin

ANS: A Patients with HIV who take protease inhibitors are susceptible to drug interactions with rifampin, which accelerates the metabolism of protease inhibitors. Rifabutin can be substituted for rifampin in patients with HIV, because the degree of acceleration of this metabolism is less. A three-drug regimen would increase drug resistance, as would a two-drug regimen.

37. A patient with a history of renal calculi has fever, flank pain, and bacteriuria. What action will the provider take? a. Prescribe antibiotic therapy after urine culture and sensitivity results are available. b. Order an prophylactic antibiotic for 6 weeks after the acute infection has cleared. c. Initiate immediate treatment with a prescription for a broad-spectrum antibiotic. d. Refer the patient for intravenous antibiotics and hospitalization.

ANS: A Patients with renal calculi are more likely to have complicated urinary tract infections that have less predictable microbiologic etiologies. Because the symptoms are mild, it is important first to obtain a culture and sensitivity to assist with antibiotic selection. If symptoms worsen, a broad-spectrum antibiotic may be started until sensitivity information is available. Intravenous antibiotics are indicated for severe pyelonephritis. Long-term prophylaxis is not indicated unless this patient develops frequent reinfection.

29. A patient will be discharged from the hospital with a prescription for TMP/SMZ. When providing teaching for this patient, the provider will discuss the importance of what action while taking this medication? a. Drinking 8 to 10 glasses of water every day b. Eating foods that are high in potassium c. Taking the medication with food d. Taking a folic acid supplement

ANS: A TMP/SMZ can injure the kidneys, because it causes deposition of sulfonamide crystals in the kidneys. Patients should be advised to drink 8 to 10 glasses of water a day to maintain a urine flow of 1200 mL in adults. Trimethoprim can cause hyperkalemia, so consuming extra potassium is unnecessary. The medication should be taken on an empty stomach. It is not necessary to consume extra folic acid, because mammalian cells use dietary folate and do not have to synthesize it; it is the process of folic acid synthesis that is altered by sulfonamides.

1. The provider working on a high-acuity medical-surgical unit is prioritizing care for four patients who were just admitted. Which patient presents with needs that the provider should address first? a. A patient with diabetes who is NPO and has a blood glucose level of 80 mg/dl needs a change in diet status after receiving 20 units of 70/30 Novolin insulin. b. A patient needs a temporary hold placed on digoxin because the heart rate was 58 beats/minute when digoxin was scheduled to be administered. c. A patient with hypertension requests an analgesic for a headache. Current blood pressure is 136/92 mm Hg. d. The patient with an allergy to penicillin who is receiving an infusion of vancomycin is concerned about the possibility of an allergic reaction.

ANS: A The NPO patient with hypoglycemia who just received 70/30 Novolin insulin takes priority, because this patient needs to consume a good source of glucose immediately or perhaps the NPO status will be discontinued for this shift. The digoxin may be withheld for the patient with a pulse of 58 beats/minute, but this is not a priority action. The patient with a headache needs to be followed up, & prescription for pain medication but because the blood pressure is 136/92 mm Hg, the headache is probably not caused by hypertension. The patient with an allergy to penicillin will not have a reaction to the vancomycin.

8. A 5-year-old child with seasonal allergies has been taking 2.5 mL of cetirizine syrup once daily. The parents tell the provider that the child does not like the syrup, and they do not think that the drug is effective. The provider will discuss which change in medication therapy? a. Cetirizine 5-mg chewable tablet once daily b. Loratadine 10-mg chewable tablet once daily c. Fexofenadine syrup 5 mL twice daily d. Desloratadine 5-mg rapid-disintegrating tablet once daily

ANS: A The child is receiving a low dose of cetirizine and can receive up to 5 mg/day in either a single dose or two divided doses. Cetirizine is available in a chewable tablet, which this child may tolerate better, so the parents should explore this option with their provider. The loratadine 10-mg chewable tablet is approved for children 6 years and older. Fexofenadine would be safe for this child, but it is unlikely that the syrup would be any better than the cetirizine syrup. Desloratadine is not approved for children under the age of 12 years.

72. A patient is HIV positive & has a previous history of drug & alcohol abuse. The patient is being treated with combination therapies, including didanosine. Which laboratory findings would most concern the provider? a. Increased serum amylase & triglycerides & decreased serum calcium b. Decreased serum amylase & serum triglycerides & increased serum calcium c. Decreased hemoglobin & hematocrit d. Increased serum amylase, decreased triglycerides, & increased platelets

ANS: A The nurse should be concerned about increased serum amylase triglycerides & a decrease in serum calcium, which are symptoms of pancreatitis, the major adverse effect of didanosine. The other laboratory test results & assessment findings are not consistent with pancreatitis & are not a concern.

5. When metronidazole is a component of the H. pylori treatment regimen, what guidance will the prescriber provide to the patient? a. Avoid any alcoholic beverages. b. Avoid foods containing tyramine. c. Take the drug on an empty stomach. d. Take the drug with food.

ANS: A The patient should be instructed to avoid alcoholic beverages, because a disulfiram-like reaction can occur if metronidazole is taken with alcohol. Nothing indicates that the patient should avoid foods containing tyramine. Metronidazole may be taken with or without food.

4. A provider considers prescribing cancer prophylaxis with tamoxifen for a 43-year-old patient with a strong family history of breast cancer. Which assessment finding will the prescriber view as a possible contraindication? a. History of deep vein thrombosis (DVT) b. Diagnosis of osteoporosis c. Current hyperlipidemia d. Prior hysterectomy

ANS: A The patient's DVT history places her at risk for thrombosis. Tamoxifen would not be indicated for this patient. A prior hysterectomy, osteoporosis, and hyperlipidemia are not contraindications to tamoxifen.

77. During a routine screening, an asymptomatic, pregnant patient at 37 weeks' gestation learns that she has an infection caused by Chlamydia trachomatis. The provider will order which drug? a. Azithromycin b. Doxycycline c. Erythromycin ethylsuccinate d. Sulfisoxazole

ANS: A The preferred treatment for C. trachomatis infection during pregnancy is either azithromycin or amoxicillin. Doxycycline can be used for nonpregnant patients. Erythromycin is used for infants. Sulfisoxazole is not recommended for pregnant women near term, because it can cause kernicterus in the infant.

4. A patient with peptic ulcer disease is otherwise healthy. The patient does not smoke and states drinking 1 or 2 glasses of wine with meals each week. Which drugs will the provider prescribe? a. Amoxicillin, clarithromycin, and omeprazole b. Amoxicillin, metronidazole, and cimetidine c. Clarithromycin, metronidazole, and omeprazole d. Tetracycline, cimetidine, and lansoprazole

ANS: A The regimen recommended for the treatment of PUD includes at least two antibiotics and an antisecretory agent. Amoxicillin, clarithromycin, and omeprazole would meet this recommendation. Patients taking metronidazole cannot consume alcohol, as this would precipitate a disulfiram-like reaction. The last option does not include two antibiotics.

6. A 2-month-old infant presents with a low-grade fever, runny nose, and severe bursts of coughing resulting in transient cyanosis. When taking a history, the provider will specifically inquire about the vaccination history for which condition? a. Pertussis b. Hepatitis B c. Measles, mumps, and rubella (MMR) d. Influenza

ANS: A This infant may have pertussis, for which the primary symptoms are low-grade fever, persistent cough, and runny nose. Infants who have not received the first set of immunizations, including the DTaP vaccine, are especially vulnerable to this disease. The hepatitis B vaccine does not protect against these symptoms. The influenza and MMR vaccines are not given to children this young.

12. A patient will be discharged home to complete treatment with intravenous cefotetan. The prescriber will include which instruction when teaching the patient about this drug treatment? a. Abstain from alcohol consumption during therapy. b. Avoid dairy products while taking this drug. c. Take an antihistamine if a rash occurs. d. Use nonsteroidal anti-inflammatory drugs (NSAIDs), not acetaminophen, for pain.

ANS: A Two cephalosporins, including cefotetan, can induce a state of alcohol intolerance and cause a disulfiram-like reaction when alcohol is consumed; therefore, patients should be advised to avoid alcohol. It is not necessary to avoid dairy products. Patients who experience a rash should report this to their provider. Cefotetan can also promote bleeding, so drugs that inhibit platelet aggregation should be avoided.

24. A patient is diagnosed with a lung infection caused by P. aeruginosa. The culture and sensitivity report shows sensitivity to all aminoglycosides. The provider knows that the rate of resistance to gentamicin is common in this hospital. What provider will order which medication? a. Amikacin b. Gentamicin c. Paromomycin d. Tobramycin

ANS: A When resistance to gentamicin and tobramycin is common, amikacin is the drug of choice for initial treatment of aminoglycoside-sensitive infections. Gentamicin would not be indicated, because resistance is more likely to develop. Paromomycin is used only for local effects within the intestine and is given orally. Tobramycin is not indicated, because organisms can more readily develop resistance.

48. A nurse is caring for a woman with breast cancer who is receiving tamoxifen. A review of this patient's chart reveals a deficiency of the CYP2D6 gene. The nurse will contact the provider to suggest: a. a different medication. b. an increased dose. c. a reduced dose. d. serum drug levels.

ANS: A Women with a deficiency of the CYP2D6 gene lack the ability to convert tamoxifen to its active form, endoxifen, & will not benefit from this drug. Another drug should be used to treat this patient's breast cancer. Increasing the dose, reducing the dose, or monitoring serum drug levels will not make this drug more effective in these women.

477. A patient has been taking levothyroxine for several years & reports that "for the past 2 weeks, the drug does not seem to work as well as before." What will the nurse do? a. Ask the patient when the prescription was last refilled. b. Expect the patient to have an elevated temperature & tachycardia. c. Suggest that the patient begin taking calcium supplements. d. Tell the patient to try taking the medication with food.

ANS: A a. Ask the patient when the prescription was last refilled. Not all levothyroxine preparations have the same drug bioavailability; therefore, if a patient is experiencing differing effects, the pharmacist may have switched brands. Asking a patient about a recent refill may help to explain why the drug has different effects. An elevated temperature & tachycardia would be signs of toxicity, not of a decrease in effectiveness. Calcium supplements & food would only interfere with absorption & further reduce the drug's effectiveness.

465. The nurse assesses a newly diagnosed patient for short-term complications of diabetes. What does this assessment include? a. Evaluation for hyperglycemia, hypoglycemia, & ketoacidosis. b. Cranial nerve testing for peripheral neuropathy. c. Pedal pulse palpation for arterial insufficiency. d. Auscultation of the carotids for bruits associated with atherosclerosis.

ANS: A a. Evaluation for hyperglycemia, hypoglycemia, & ketoacidosis. High blood sugar, low blood sugar, & ketoacidosis are short-term complications of diabetes. Microvascular & macrovascular complications, such as peripheral neuropathy, are long-term complications of diabetes. Arterial insufficiency & atherosclerosis also are long-term complications of diabetes.

463. Which statement is correct about the contrast between acarbose & miglitol? a. Miglitol has not been associated with hepatic dysfunction. b. With miglitol, sucrose can be used to treat hypoglycemia. c. Miglitol is less effective in African Americans. d. Miglitol has no gastrointestinal side effects.

ANS: A a. Miglitol has not been associated with hepatic dysfunction. Unlike acarbose, miglitol has not been associated with hepatic dysfunction. Sucrose should not be used to treat hypoglycemia with miglitol. Miglitol is more effective in African-American patients. Miglitol has gastrointestinal side effects.

469. A patient with type 1 diabetes reports mixing NPH and regular insulin to allow for one injection. What should the nurse tell the patient? a. This is an acceptable practice. b. These two forms of insulin are not compatible & cannot be mixed. c. Mixing these two forms of insulin may increase the overall potency of the products. d. NPH insulin should only be mixed with insulin glargine.

ANS: A a. This is an acceptable practice. NPH insulin is the only insulin suitable for mixing with short-acting insulins, such as insulin aspart [NovoLog]. These insulins are compatible & are mixed frequently for management of diabetics. The overall potency of each insulin is not increased by mixing them. Insulin glargine cannot be mixed with any other insulin for administration.

68. A patient who is taking nelfinavir reports moderate to severe diarrhea. What action will the provider recommend? a. An over-the-counter antidiarrheal drug b. Immediate discontinuation of the nelfinavir c. Reducing the dose of nelfinavir by half d. Taking the nelfinavir with food to avoid side effects

ANS: A A dose-limiting effect of nelfinavir is moderate to severe diarrhea, which can be managed with OTC antidiarrheal medications. Unless the symptoms become severe, withdrawing the nelfinavir is not indicated. Reducing the dose by half or taking it with food is not indicated.

26. A patient who has been receiving intravenous gentamicin for several days reports having had a headache for 2 days. What action will the provider take? a. Discontinue the gentamicin. b. Order a gentamicin trough before the next dose is given. c. Prescribe an analgesic to control headache discomfort. d. Order renal function tests to evaluate for potential nephrotoxicity.

ANS: A A persistent headache may be a sign of developing ototoxicity, and since ototoxicity is largely irreversible, gentamicin should be withdrawn at the first sign of developing ototoxicity. A gentamicin trough should be obtained before the next dose is given when high gentamicin levels are suspected. Analgesics are not indicated until a serious cause of the headache has been ruled out. A headache is an early sign of ototoxicity, not nephrotoxicity.

51. A nurse is teaching parents how to use an Epi-Pen for their child, who has a peanut allergy. Which statement by the parents indicates understanding of the teaching? a. "After using the Epi-Pen, we must go to the emergency department." b. "The Epi-Pen should be stored in the refrigerator, because epinephrine is sensitive to heat." c. "The teacher should call us when symptoms start so that we can bring the Epi-Pen to school." d. "We should jab the device into the thigh until it is empty of solution."

ANS: A After using the Epi-Pen, it is important that the individual get medical attention as quickly as possible. The effects of epinephrine fade in 10 to 20 minutes, & the anaphylactic reaction can be biphasic & prolonged. Epinephrine is sensitive to heat, but storing the device in the refrigerator can compromise the injection mechanism; the device should be stored at room temperature in a dark place. Individuals who need an Epi-Pen must have the device with them at all times; any delay in treatment can be fatal, because anaphylaxis can occur within minutes after exposure. The Epi- Pen contains 2 mL of epinephrine, but only 0.3 mL is injected; the device will not be empty with a successful injection.

10. A patient who takes allopurinol for chronic gout develops a rash. Vital signs are within normal limits and there are no signs of patient distress. What action will the provider take? a. Hold the allopurinol and confirm an existing drug reaction. b. Discontinue the allopurinol and prescribe colchicine. c. Prescribe an antihistamine to minimize the drug side effects. d. Lower the dose of allopurinol.

ANS: A Allopurinol can cause a hypersensitivity syndrome, which is characterized by a rash and fever. If these occur, the drug should be discontinued immediately. Antihistamines are not indicated. Lowering the dose of allopurinol is not indicated. It is inappropriate to replace allopurinol with colchicine as these are prescribed for different purposes.

4. A 14-year-old male patient who has not yet begun puberty reports being bullied at school. The boy's father reports that he and his father also experienced teasing as adolescents because they did not begin puberty until around age 16 years. What will the provider tell the patient and his father when they ask what can be done? a. "A limited course of androgen therapy may be prescribed, but it is not necessary." b. "He will eventually begin puberty, so this is nothing to worry about." c. "There is nothing that can be done about the puberty delay, but I can make a referral for psychological counseling." d. "The risk of accelerated growth plate closure is too great to warrant androgen therapy."

ANS: A Although treatment is not required in this patient, the psychological effects of delayed puberty indicate a limited course of androgen therapy. Telling the parents not to worry does not address their concerns. This patient will not be a candidate for long-term androgen replacement therapy. Limited treatment can minimize the risk of epiphyseal closure, especially with close monitoring and radiographs.

2. A patient has been diagnosed with fungal otitis externa (otomycosis). What will the provider prescribe initially? a. Acidifying ear drops for 1 week b. Antifungal ear drops for 14 days c. Intravenous (IV) antifungal agents d. Oral antibiotics for 7 days

ANS: A As a rule, otomycosis can be managed with thorough cleansing & application of acidifying drops (2% acetic acid solution applied three or four times a day for 7 days). If that does not work, an antifungal drug solution can be tried. If the infection fails to respond to the drug, oral antifungal therapy may be needed. Neither antibiotic ear drops nor oral antibiotics are indicated as the first course of treatment. IV antifungal agents are not indicated at all.

8. A patient with refractory gout will be admitted for treatment with intravenous pegloticase. What additional action will the prescriber take to manage the patient's treatment? a. Prescribe an antihistamine and a glucocorticoid to be administered prior to beginning the pegloticase infusion. b. Initiate allopurinol therapy 24 hours prior to treatment with pegloticase. c. Screen the patient for folic acid deficiency. d. Prescribe a short-acting bronchodilator to be used as needed.

ANS: A Because pegloticase poses a risk of triggering anaphylaxis, patients should be pretreated with an antihistamine and a glucocorticoid. Colchicine or NSAIDs are indicated at the initiation of treatment with pegloticase to reduce the intensity of gout flare-ups. There is no reason to prescribe allopurinol 24 hours prior to pegloticase. Bronchodilators are not indicated. Patients should be screened for glucose-6-phosphate dehydrogenase (G6PD) deficiency before receiving the drug.

52. A patient who is being treated with medication for both HIV and tuberculosis but a four-drug regimen taken for 3 months has shown no improvement in symptoms. Which drug will the provider add to this patient's regimen? a. Bedaquiline b. Capreomycin c. Ethionamide d. Pyridoxine

ANS: A Bedaquiline is a newer, highly effective anti-TB drug that does not accelerate the metabolism of HIV drugs and is sued for multidrug-resistant TB. Capreomycin is a second-line drug used for drug-resistant TB, but is not as effective as bedaquiline. Ethionamide is a second-line drug that is less well tolerated of all anti-TB agents and is used only when there is no alternative. Pyridoxine is given to prevent peripheral neuritis in patients taking isoniazid.

16. A patient with open-angle glaucoma has a history of chronic obstructive pulmonary disease. Which ophthalmic β blocker will the provider prescribe? a. Betaxolol b. Timolol c. Levobunolol d. Metipranolol

ANS: A Betaxolol is the only ophthalmic β blocker that is β1 selective, meaning that it has less chance of causing bronchial constriction. It is preferred for patients with asthma & COPD. The other agents are not β1 selective.

54. A patient who is receiving intravenous ciprofloxacin for pneumonia develops diarrhea. A stool culture is positive for Clostridium difficile. What action will the provider take? a. Prescribe metronidazole. b. Increase the dose of ciprofloxacin. c. Restrict dairy products. d. Switch to gemifloxacin.

ANS: A C. difficile is resistant to fluoroquinolones; metronidazole is the drug of choice to treat this infection. Metronidazole is lethal only to anaerobic organisms, so the ciprofloxacin should be continued to treat the pneumonia. Increasing the dose of ciprofloxacin is not indicated, because C. difficile is resistant to ciprofloxacin. Gemifloxacin is approved for use in respiratory infections. Dairy consumption will have little effect on the diarrhea.

17. Which side effect of clindamycin causes the provider the most concern and may warrant discontinuation of the drug? a. Diarrhea b. Headache c. Nausea d. Vomiting

ANS: A Clostridium difficile-associated diarrhea (CDAD) is a serious, sometimes fatal suprainfection associated with clindamycin. Patients with diarrhea should notify their prescriber immediately and discontinue the drug until this condition has been ruled out. Headache, nausea, and vomiting do not warrant discontinuation of the drug and are not associated with severe side effects.

3. A patient newly diagnosed with peptic ulcer disease (PUD) reports taking low-dose aspirin (ASA) for prevention of cardiovascular disease. The medical history determines that the patient drinks 2 to 3 cups of coffee each day, eats three meals a day, and has a glass of wine with dinner 3 or 4 nights per week. The provider will counsel this patient to what lifestyle action? a. Change the meal pattern to five or six smaller meals per day. b. Discontinue taking aspirin, because it can irritate the stomach. c. Stop drinking wine or any other alcoholic beverage. d. Switch to a decaffeinated coffee and reduce the number of servings.

ANS: A Consumption of five or six smaller meals a day can reduce fluctuations in the intragastric pH, which may facilitate recovery. ASA should be avoided, along with other NSAIDs except for low-dose ASA used for the prevention of cardiovascular disease. No hard data implicate moderate alcohol intake as a contributor to PUD. No data indicate that caffeine contributes to PUD.

55. A provider would prescribe which antibiotic to a patient diagnosed with methicillin-resistant Staphylococcus aureus (MRSA)? a. Daptomycin b. Levofloxacin c. Norfloxacin d. Ciprofloxacin

ANS: A Daptomycin is active against MRSA. Levofloxacin and norfloxacin are not approved to treat MRSA. Ciprofloxacin is a poor choice for staphylococcal infections, including MRSA.

4. An adolescent had a serum glucose test at a health fair. The parent calls the clinic and says, "The level was 125 mg/dL. Does that mean my child has diabetes?" What is the provider's most accurate response? a. "Unless your child were fasting for longer than 8 hours, this does not necessarily indicate diabetes." b. "At this level, there is a possibility of diabetes. We'll schedule your child for an oral glucose tolerance test this week." c. "This level is conclusive evidence that your child has diabetes." d. "This level is conclusive evidence that your child do not have diabetes."

ANS: A If a person has not fasted for 8 hours, a blood sugar level of 125 mg/dL would be considered normal, because it is less than 200 mg/dL for a random sampling. Also, a person must have positive outcomes on two separate days to be diagnosed with diabetes. This patient does not need to have an oral glucose tolerance test, because the 125 mg/dL reading is so far below 200 mg/dL, which would require further work-up. No conclusive evidence indicates that this patient has diabetes, because the random sample value is so low, and the patient has not had two separate tests on different days. However, this also is not conclusive evidence that the patient does not have diabetes.

19. A patient who uses latanoprost ophthalmic drops tells the provider, "My eyes used to be greenish-brown, but now they're brown." What response will the provider give to address the patient's concern? a. Reassure the patient that eye-color change is a harmless side effect. b. Refer the patient to an ophthalmologist. c. Explain that the eye color is unchanged but color perception is a drug side effect. d. Tell the patient that this effect will reverse when the medication is withdrawn.

ANS: A Latanoprost can cause a harmless, heightened brown pigment of the iris that stops when the medication is discontinued but does not regress or reverse. It is not a sign of a toxic reaction. Color perception does not occur.

12. Which new diagnosis would prompt the provider to discontinue a PRN order for magnesium hydroxide? a. Renal failure b. Cirrhosis c. Hemorrhoids d. Prostatitis

ANS: A Magnesium can accumulate to toxic levels in patients with renal dysfunction. Magnesium hydroxide is not contraindicated for patients with hemorrhoids, prostatitis, or cirrhosis.

15. To provide effective protection against the full range of ultraviolet (UV) radiation, an organic sunscreen must contain which agent? a. Avobenzone b. para-Aminobenzoic acid (PABA) c. Titanium dioxide d. Zinc oxide

ANS: A Only one organic sunscreen, avobenzone, absorbs the full range of UV radiation. The other agents do not protect against the full range of UV radiation. Titanium dioxide & zinc oxide are inorganic screens.

51. A patient is about to begin therapy with ethambutol. Before initiating treatment with this drug, the provider orders which test(s) to determine an accurate baseline? a. Color vision and visual acuity b. Complete blood cell (CBC) count c. Hearing testing and a tympanogram d. Hepatic function tests

ANS: A Optic neuritis is a dose-related adverse effect of ethambutol. Patients receiving this drug should have color vision and visual acuity testing before therapy starts and periodically thereafter. A CBC, hearing evaluations, and hepatic function testing are not recommended.

58. The nurse is preparing to administer amphotericin B intravenously. The provider will pretreat the patient with which medications? a. Acetaminophen and diphenhydramine b. Aspirin and diphenhydramine c. Ibuprofen and diphenhydramine d. Morphine sulfate and acetaminophen

ANS: A Optimum pretreatment before the administration of amphotericin B comprises acetaminophen and diphenhydramine [Motrin] is not suggested as pretreatment. Aspirin is an option, but it may increase kidney damage. Morphine is not indicated in the pretreatment regimen.

13. An older adult patient takes an opioid analgesic after a right hip open reduction internal fixation (ORIF). Which medication will the provider prescribe to prevent constipation? a. Docusate sodium b. GoLYTELY c. Glycerin suppositories d. Polyethylene glycol

ANS: A Oxycodone can be constipating. The patient needs something prophylactically, such as docusate sodium. In addition, the patient's mobility is limited, which can further increase the risk of constipation. GoLYTELY is not indicated for constipation; it is used for cleansing the bowel before diagnostic procedures. Glycerin suppositories and polyethylene glycol are indicated for treatment of occasional constipation but not for prophylaxis.

18. A patient with glaucoma will begin using brimonidine for long-term reduction of increased ocular pressure. The medication education provided by the prescriber appears to have been effective when the patient makes which statement? a. "After using the drops, I should wait 15 minutes before putting in contacts." b. "Because this is a topical medication, drowsiness will not occur." c. "My blood pressure may increase when using this medication." d. "If my eyes begin to itch or turn red, it means I am allergic to this drug."

ANS: A Patients using this drug should wait 15 minutes before putting in contacts, because soft contacts can absorb the drug. Even though the medication is topical, it can be absorbed systemically, causing systemic side effects such as drowsiness or lowered blood pressure. Itching & hyperemia may occur & do not indicate allergy.

38. A patient has a positive urine culture 1 week after completion of a 3-day course of antibiotics. What action will the prescriber take? a. Prescribe a 2-week course of antibiotics. b. Order tests to evaluate for a structural abnormality of the urinary tract. c. Prescribe long-term prophylaxis with low-dose antibiotics. d. Treat the patient with intravenous antibiotics.

ANS: A Patients who develop a subsequent urinary tract infection after treatment are treated in a stepwise fashion, beginning with a longer course of antibiotics. The next steps would be to begin a 4- to 6-week course of therapy, followed by a 6-month course of therapy if that is unsuccessful. If urinary tract infections are thought to be caused by other complicating factors, an evaluation for structural abnormalities may be warranted. Unless the infections are severe or are complicated, intravenous antibiotics are not indicated.

49. A patient who is being treated for HIV infection has a 5-mm area of induration after a routine TB skin test. The patient's chest radiograph is normal, and there are no other physical findings. The provider will prescribe which medications to begin treatment? a. Isoniazid and rifabutin b. Isoniazid and rifampin c. Isoniazid and rifapentine d. Isoniazid and pyrazinamide

ANS: A Rifabutin is used off-label as an alternative to rifampin to treat TB in patients with HIV, because it has less impact on the metabolism of protease inhibitors. The effects of rifapentine on protease inhibitors are similar to those of rifampin. Pyrazinamide is not indicated.

4. Amoxicillin is prescribed for a 12-month-old child who developed a second middle ear infection since age 8 months. Three days later, the parent calls to report that the child continues to have a temperature of 39.5°C & is unable to sleep well because of pain. What action will the provider take to provide effective care to this child? a. Discuss prescribing amoxicillin/clavulanate with the child's parents. b. Prescribe a medication that promotes sleep. c. Advise continuing the amoxicillin as ordered & provide a schedule of prn dosing for ibuprofen for pain. d. Discuss with the parent the child's need for probably surgery for tympanostomy tubes placement to reduce infections.

ANS: A The child displays evidence of resistant acute otitis media (AOM). Resistance is treated with high-dose amoxicillin/clavulanate. The high dose of amoxicillin increases activity against amoxicillin-resistant S. pneumoniae, & the clavulanate component overcomes β-lactam resistance of H. influenzae & M. catarrhalis. Prescribing a sleep aid does not address the underlying problem. Recommending a drug for pain is appropriate, but the antibiotic needs to be changed. IM Rocephin is not recommended. Surgery for insertion of bilateral tympanostomy tubes (BMTTs) is used to reduce the number of episodes in children with recurrent AOM that occurs three or more times within 6 months or four or more times in a year.

41. A nurse is preparing to give an antibiotic to a patient who reports being allergic to antibiotics. Before giving the medication, what will the nurse do first? a. Ask whether the patient has taken this antibiotic for other infections b. Question the patient about allergies to other medications c. Request an order for a lower dose of the antibiotic d. Request an order for an antihistamine

ANS: A The nurse needs to assess whether the patient is truly allergic to this drug. Allergic reactions require previous exposure to the drug, so the nurse should ask whether the patient has taken this antibiotic before. If a patient is allergic to a drug, lowering the dose will not decrease the risk of allergic reaction. Antihistamines sometimes are given when patients must take a drug to which they are allergic.

50. A patient comes to the clinic and receives valacyclovir for a herpes-zoster virus. When will the provider instruct the patient to take the medication? a. Without regard to meals b. Without any dairy products c. Each morning d. On an empty stomach

ANS: A The patient may take the medication without regard to meals. The patient does not need to avoid dairy products, take the pill only in the morning, or take it on an empty stomach.

35. A 20-year-old female patient presents with suprapubic discomfort, pyuria, dysuria, and bacteriuria greater than 100,000/mL of urine. Which are the most likely diagnosis and treatment? a. Uncomplicated lower urinary tract infection treatable with short-course therapy b. Complicated lower urinary tract infection treatable with single-dose therapy c. Uncomplicated upper urinary tract infection requiring 14 days of oral antibiotics d. Complicated upper urinary tract infection requiring parenteral antibiotics

ANS: A These are symptoms of uncomplicated cystitis, which is a lower urinary tract infection that can be treated with a short course of antibiotics. Short-course therapy is more effective than single-dose therapy and is preferred. A complicated lower urinary tract infection would be associated with some predisposing factor, such as renal calculi, an obstruction to the flow of urine, or an indwelling catheter. Upper urinary tract infections often include severe flank pain, fever, and chills.

20. A provider considers prescribing timolol for a patient with primary open-angle glaucoma. Which condition could be worsened if this drug is prescribed? a. Asthma b. Hypertension c. Migraine headaches d. Supraventricular tachycardia

ANS: A Timolol is a β blocker & can precipitate bronchoconstriction if absorbed systemically in sufficient amounts. Patients with asthma may develop shortness of breath & wheezing. Systemically absorbed β blockers can have beneficial, not detrimental, effects on hypertension, tachycardia, & migraine headaches.

56. Which condition is generally treated with oral antifungal agents? a. Tinea capitis b. Tinea corporis c. Tinea cruris d. Tinea pedis

ANS: A Tinea capitis must be treated with oral agents for 6 to 8 weeks. Tinea corporis, tinea cruris, and tinea pedis may be treated topically.

12. A patient receiving intracavernousal alprostadil discusses switching to the transurethral administration route. Which information does the provider include as a benefit of changing administration routes? a. Transurethral administration has fewer side effects. b. Transurethral administration helps increase arterial flow to the penis. c. Erection develops in about 30 minutes and lasts up to 2 hours with transurethral administration. d. The required dose is much smaller with transurethral administration.

ANS: A Transurethral administration has fewer side effects than intracavernousal administration of alprostadil. Both intracavernousal and transurethral administration lead to increased arterial blood flow to the penis. The dosage of transurethral alprostadil ranges from 125 to 1000 mcg, which is higher than the dose required for intracavernousal administration (5 to 40 mcg). Erection develops 5 to 10 minutes after drug insertion and lasts 30 to 60 minutes.

4. A child with an ear infection is not responding to treatment with amoxicillin. What alternative medication will the provider order? a. Amoxicillin-clavulanic acid b. Ampicillin c. Nafcillin d. Penicillin G

ANS: A β -lactamase inhibitors are drugs that inhibit bacterial β-lactamases. These drugs are always given in combination with a penicillinase-sensitive penicillin. Augmentin contains amoxicillin and clavulanic acid and is often used when patients fail to respond to amoxicillin alone. Ampicillin is similar to amoxicillin, but amoxicillin is preferred and, if drug resistance occurs, ampicillin is equally ineffective. Pharmaceutical chemists have developed a group of penicillins that are resistant to inactivation by β-lactamases (e.g., nafcillin), but these drugs are indicated only for penicillinase-producing strains of staphylococci. Penicillin G would be as ineffective as amoxicillin if β-lactamase is present.

472. An adolescent patient recently attended a health fair & had a serum glucose test. The patient telephones the nurse & says, "My level was 125 mg/dL. Does that mean I have diabetes?" What is the nurse's most accurate response? a. "Unless you were fasting for longer than 8 hours, this does not necessarily mean you have diabetes." b. "At this level, you probably have diabetes. You will need an oral glucose tolerance test this week." c. "This level is conclusive evidence that you have diabetes." d. "This level is conclusive evidence that you do not have diabetes."

ANS: A a. "Unless you were fasting for longer than 8 hours, this does not necessarily mean you have diabetes." If a person has not fasted for 8 hours, a blood sugar level of 125 mg/dL would be considered normal, because it is less than 200 mg/dL for a random sampling. Also, a person must have positive outcomes on two separate days to be diagnosed with diabetes. This patient does not need to have an oral glucose tolerance test, because the 125 mg/dL reading is so far below 200 mg/dL, which would require further work-up. No conclusive evidence indicates that this patient has diabetes, because the random sample value is so low, & the patient has not had two separate tests on different days. However, this also is not conclusive evidence that the patient does not have diabetes.

461. The nurse working on a high-acuity medical-surgical unit is prioritizing care for four patients who were just admitted. Which patient should the nurse assess first? a. The NPO patient with a blood glucose level of 80 mg/dL who just received 20 units of 70/30 Novolin insulin. b. The patient with a pulse of 58 beats/minute who is about to receive digoxin [Lanoxin]. c. The patient with a blood pressure of 136/92 mm Hg who complains of having a headache. d. The patient with an allergy to penicillin who is receiving an infusion of vancomycin [Vancocin].

ANS: A a. The NPO patient with a blood glucose level of 80 mg/dL who just received 20 units of 70/30 Novolin insulin. The NPO patient with hypoglycemia who just received 70/30 Novolin insulin takes priority, because this patient needs to consume a good source of glucose immediately or perhaps the NPO status will be discontinued for this shift. The digoxin may be withheld for the patient with a pulse of 58 beats/minute, but this is not a priority action. The patient with a headache needs to be followed up, but because the blood pressure is 136/92 mm Hg, the headache is probably not caused by hypertension. The patient with an allergy to penicillin will not have a reaction to the vancomycin.

15. A provider considers prescribing tamoxifen for a woman with breast cancer. Upon reviewing results of genetic testing, the prescriber notes that the patient has variations in the CYP2Dy allele resulting in a deficiency of the CYP2D6 isoenzymes. What action will this deficiency warrant in the prescribing of tamoxifen, a CYP2D6 substrate? a. The tamoxifen will not be prescribed. b. The individual doses of tamoxifen will be increased. c. The tamoxifen will be ordered but in lower than normal dosage. d. The patient's serum tamoxifen level will be routinely monitored.

ANS: A a. The tamoxifen will not be prescribed. Women with a deficiency of CYP2D6 isoenzymes lack the ability to convert tamoxifen to its active form, endoxifen, & will not benefit from this drug. Another drug should be used to treat this patient's breast cancer. Increasing the dose, reducing the dose, or monitoring serum drug levels will not make this drug more effective in these women.

19. A patient recently diagnosed with HIV is prescribed several medications to treat the condition. Which factors could impact the patient's adherence to the treatment regimen? Select all that apply. a. The patient is uninsured b. The patient works three part-time jobs c. The medication regimen includes six different pills d. Patient has an eighth-grade reading comprehension level e. Medication regimen requires medication be taken at regular 4-hour intervals.

ANS: A , B , C , E Lack of insurance coverage can inhibit the patient from purchasing the medications, limiting his access to treatment. Having three part-time jobs indicates that the patient has a busy schedule, which contributes to forgetfulness & poor adherence. The more complex the medication regimen, the more difficult it is to maintain patient adherence. Although a patient with an eighth-grade reading comprehension level may have difficulty understanding professional medical language, medication teaching can be adjusted to meet the patient's learning needs

MULTIPLE RESPONSE 1. A provider determines that a patient requires a prescription for a narrow spectrum antibacterial drug to treat a gram-positive cocci-related infection. What option will the provider consider? (Select all that apply.) a. Erythromycin b. Penicillin G c. Tetracycline d. Vancomycin e. Trimethoprin

ANS: A, B, D Examples of narrow spectrum antibacterial drugs include penicillin G, erythromycin, & vancomycin. Trimethoprin & tetracycline are broad spectrum antibiotics.

2. What medication will the provider prescribe for a patient with an Enterococcus faecium associated infection who has proven to be ampicillin resistant? (Select all that apply.) a. Linezolid b. Vancomycin c. Tigecycline d. Carbapenems

ANS: A, C An alternative treatment for an enterococcus faecium-related infection diagnosed in a patient who is ampicillin resistant includes linezolid. None of the other options are known to be effective in the case of an enterococcus faecium infection.

MULTIPLE RESPONSE 1. Which conditions may be treated with monoclonal antibody (mAb) therapy? (Select all that apply.) a. Breast cancer b. Heart murmur c. Hemophilia d. Asthma e. Migraine headaches

ANS: A, C, D, E MAbs have many uses, including treatment of cancer, asthma, hemophilia, & prevention of migraine headache. This medication is not appropriate for treatment of a heart murmur.

5. Which statement correctly informs a provider's decision regarding whether to order an antiestrogen drug or an aromatase inhibitor for treatment of breast cancer. a. Antiestrogen drugs decrease the risk for thromboembolic events. b. Antiestrogen drugs increase the risk for endometrial cancer. c. Aromatase inhibitors block the production of estrogen by the ovaries. d. Aromatase inhibitors can be used for tumor cells that lack estrogen receptors

ANS: B Antiestrogen drugs cause proliferation of endometrial tissue by acting as receptor agonists at receptors in the uterus. Antiestrogen drugs increase the risk of thromboembolic events. Aromatase inhibitors block the production of estrogen from androgenic precursors, not by the ovaries. Aromatase inhibitors are used to treat ER-positive breast cancer.

9. An adolescent with acne who uses benzoyl peroxide lotion twice daily reports experiencing drying & burning of the skin. What action will the prescriber take? a. Recommend applying lotion to the skin after applying the drug. b. Advise the patient to reduce the frequency to one application a day. c. Discontinue the medication to prevent further localized allergic reaction. d. Prescribe a gel formulation of the drug.

ANS: B Benzoyl peroxide may cause drying & peeling of the skin. If signs of severe local irritation occur, such as burning or blistering, the frequency of application should be reduced. Applying lotion is not indicated. These symptoms are not consistent with an allergic reaction. There is no difference in skin reactions between the gel & the lotion formulations.

18. When ordering castor oil for a patient, what guidance will the provider share with the nurse? a. Administer the medication at bedtime. b. Chill the medication and mix it with fruit juice. c. Provide teaching about home use of this medication. d. Teach the patient that the effects will occur slowly.

ANS: B Castor oil has an unpleasant taste that can be improved by chilling it and mixing it with fruit juice. The medication acts quickly and should not be given at bedtime. It is only used when prompt evacuation of intestinal contents is needed, as for radiological procedures, so the patient will not be instructed in home use of the medication and should be taught that the effects will be rapid.

12. A patient who will begin taking colchicine for gout currently takes ibuprofen, simvastatin, amoxicillin, and digoxin. What action will the provider take to manage this medication regime? a. Prescribe a different antibiotic. b. Discuss the potential risk of muscle injury. c. Order cardiorespiratory monitoring. d. Suggest that the patient discontinue the ibuprofen.

ANS: B Colchicine can cause rhabdomyolysis, and this risk is increased in patients who also take simvastatin or other statin drugs. Amoxicillin does not interact with colchicine. The side effects of digoxin are not increased by concurrent use with colchicine. NSAIDs such as ibuprofen can safely be taken with colchicine.

24. A provider discusses the use of immunosuppressants for the treatment of inflammatory bowel disease (IBD) with a patient. Which statement by the patient indicates understanding of the teaching? a. "Azathioprine helps induce rapid remission of IBD." b. "Cyclosporine can be used to induce remission of IBD." c. "Cyclosporine does not have serious adverse effects." d. "Methotrexate is used long term to maintain remission of IBD."

ANS: B Cyclosporine can be given intravenously to induce rapid remission of IBD. Azathioprine has delayed onset of effects up to 6 months and is not used to induce rapid remission. Cyclosporine is a toxic compound that can cause renal impairment, neurotoxicity, and immune suppression. Methotrexate is used to promote short-term remission.

5. An infant has allergies & often develops a pruritic rash when exposed to allergens. The infant's parents ask the nurse about using a topical antihistamine. What should the nurse tell them? a. Antihistamines given by this route are not absorbed as well in children. b. Applying an antihistamine to the skin can cause toxicity in this age group. c. The child will also need oral medication to achieve effective results. d. Topical medications have fewer side effects than those given by other routes.

ANS: B Drug absorption through the skin is more rapid in infants, because their skin is thinner & has greater blood flow; therefore, infants are at increased risk of toxicity from topical drugs. Because of increased drug absorption through the skin, infants should not be given additional drugs via other routes. If a drug is more likely to be absorbed rapidly, it will have more side effects.

52. A nursing student asks the nurse why epinephrine, & not other adrenergic agonists, is used to treat anaphylactic shock. What will the nurse tell the student? a. "Epinephrine is the only adrenergic agonist that may be given parenterally." b. "Epinephrine has the ability to activate multiple types of adrenergic receptors." c. "Other adrenergic agonists have more severe adverse effects & are not safe in the doses needed to treat anaphylaxis." d. "Other adrenergic agonists have little or no effects on beta2-adrenergic receptors."

ANS: B Epinephrine is used to treat anaphylactic shock because of its ability to activate multiple adrenergic receptor types. Activation of beta1 receptors helps to increase cardiac output & improve blood pressure as well as suppress glottal edema. Activation of beta2 receptors helps to counteract bronchoconstriction. Activation of alpha1 receptors also causes vasoconstriction, which improves blood pressure. Isoproterenol may also be given parenterally but does not activate multiple receptor types. Other adrenergic agonists, such as albuterol, are more specific to beta2 receptors & have fewer side effects.

24. A patient will begin taking fexofenadine for hay fever. What information will the provider include in patient teaching? a. Fexofenadine should be taken with food to prevent gastrointestinal symptoms. b. The medication may be taken once or twice daily. c. Tolerance to sedation will occur in a few weeks. d. With renal impairment, this drug should be taken every other day.

ANS: B Fexofenadine may be given 60 mg twice daily or 180 mg once daily. Fexofenadine does not need to be given with food. Sedation is not a common side effect of fexofenadine. There is no caution to reduce the dosage or increase the dosing interval in patients with renal impairment who take fexofenadine.

62. A patient with a history of congestive heart failure and renal impairment is diagnosed with esophageal candidiasis. Which antifungal agent will the provider prescribe this patient? a. Amphotericin B b. Fluconazole c. Itraconazole d. Voriconazole

ANS: B Fluconazole is a drug of choice for treating systemic candidal infections. Amphotericin is nephrotoxic and should not be used in patients with existing renal disease. Itraconazole is a possible alternative agent for treating candidiasis but has serious cardiac side effects. Voriconazole is a drug of first choice for treating aspergillosis but not for candidiasis.

64. A patient with HIV contracts herpes simplex virus (HSV), and the prescriber orders acyclovir 400 mg PO twice daily for 10 days. After 7 days of therapy, the patient reports having an increased number of lesions. What action will the provider take? a. Extend this patient's drug therapy to twice daily for 12 months. b. Order intravenous foscarnet every 8 hours for 2 to 3 weeks. c. Increase the acyclovir dose to 800 mg PO five times daily. d. Order intravenous valacyclovir [Valtrex] 1 g PO twice daily for 10 days.

ANS: B Foscarnet is active against all known herpesviruses and is used in immunocompromised patients with acyclovir-resistant HSV or VZV. This patient is demonstrating resistance to acyclovir, so extending acyclovir therapy or increasing the acyclovir dose will not be effective. Valacyclovir is not approved for use in immunocompromised patients because of the risk for thrombotic thrombocytopenic purpura/hemolytic uremic syndrome.

13. A patient reporting burning on urination and increased frequency has a history of frequent urinary tract infections (UTIs). The patient is going out of town in 2 days and the provider wants to treat the infection quickly. What medication will the provider order? a. Aztreonam b. Fosfomycin c. Trimethoprim/sulfamethoxazole d. Vancomycin

ANS: B Fosfomycin has been approved for single-dose therapy of UTIs in women. Vancomycin and aztreonam are not indicated for UTIs. Bactrim is indicated for UTIs, but administration of a single dose is not therapeutic.

45. A patient comes to a clinic for tuberculosis medications 2 weeks after beginning treatment with a four-drug induction phase. The patient's sputum culture remains positive, and no drug resistance is noted. At this point, the provider will take what action? a. Change the regimen to a two-drug continuation phase. b. Continue the four-drug regimen and recheck the sputum in 2 weeks. c. Obtain a chest radiograph and consider adding another drug to the regimen. d. Question the patient about adherence to the drug regimen.

ANS: B In patients with positive pretreatment sputum test results, sputum should be evaluated every 2 to 4 weeks until cultures are negative and then monthly thereafter. In the absence of drug resistance, treatment with the same regimen should continue. Sputum cultures should become negative in over 90% of patients in 3 or more months. The induction phase should last 2 months, so this patient should remain on a four-drug regimen. It is not necessary to order a chest radiograph or to add another drug at this stage of treatment. The patient is stable and has not developed symptoms that cause concern, so the patient does not need to be questioned about adherence.

482. The nurse is caring for a pregnant patient recently diagnosed with hypothyroidism. The patient tells the nurse she does not want to take medications while she is pregnant. What will the nurse explain to this patient? a. Hypothyroidism is a normal effect of pregnancy & usually is of no consequence. b. Neuropsychologic deficits in the fetus can occur if the condition is not treated. c. No danger to the fetus exists until the third trimester. d. Treatment is required only if the patient is experiencing symptoms.

ANS: B Maternal hypothyroidism can result in permanent neuropsychologic deficits in the child. Hypothyroidism is not a normal effect of pregnancy & is a serious condition that can affect both mother & fetus. The greatest danger to the fetus occurs in the first trimester, because the thyroid does not fully develop until the second trimester. Early identification is essential. Symptoms often are vague. Treatment should begin as soon as possible, or mental retardation & other developmental problems may occur.

11. After being educated about administration of papaverine plus phentolamine for erectile dysfunction, which statement made by the patient indicates a need for further teaching by the prescriber? a. "I can expect rapid onset of an erection." b. "I will take the pill 30 minutes before sex." c. "I should get up from my seat slowly when taking this." d. "I will notify the provider if I feel hard areas on my penis."

ANS: B Papaverine plus phentolamine is administered as an intracavernousal injection, not as an oral pill. The patient should be educated about the correct route of administration. Desired effect may be achieved quickly with administration. Adverse effects of administration include orthostatic hypotension and fibrotic nodules in the corpus cavernosum from injection. The patient should be educated about these effects and ways to prevent complications, such as rising slowly from a seated position to prevent dizziness.

10. A patient receiving a cephalosporin develops a secondary intestinal infection caused by Clostridium difficile. What action will the provider take to provide effective care for this patient? a. Adding an additional antibiotic to the patient's regimen b. Discontinuing the cephalosporin and beginning metronidazole c. Discontinuing all antibiotics and providing fluid replacement d. Increasing the dose of the cephalosporin and providing isolation measures

ANS: B Patients who develop C. difficile infection (CDI) as a result of taking cephalosporins or other antibiotics need to stop taking the antibiotic in question and begin taking either metronidazole or vancomycin. Adding one of these antibiotics without withdrawing the cephalosporin is not indicated. CDI must be treated with an appropriate antibiotic, so stopping all antibiotics is incorrect. Increasing the cephalosporin dose would only aggravate the CDI.

1. An older adult patient with a history of chronic obstructive pulmonary disease (COPD) develops bronchitis. The patient has a temperature of 39.5°C. What action will the provider initially take to assure effective care for this patient? a. Order a sputum culture and prescribe an antibiotic based on the results. b. Order an empiric antibiotic while waiting for sputum culture results. c. Treat symptomatically, because antibiotics are usually ineffective against bronchitis. d. Treat the patient with more than one antibiotic without obtaining cultures.

ANS: B Patients with severe infections should be treated while culture results are pending. If a patient has a severe infection or is at risk of serious sequelae if treatment is not begun immediately, it is not correct to wait for culture results before beginning treatment. Until a bacterial infection is ruled out, treating symptomatically is not indicated. Treating without obtaining cultures is not recommended.

7. A provider prescribes the keratolytic agent salicylic acid for a patient with extensive acne on the face, shoulders, trunk, & back. What drug information does the provider give to the patient? a. "Peeling & drying are desired effects of this drug." b. "Call the clinic if you develop nausea, vomiting, or ringing in the ears." c. "Tinnitus is a common side effect of little concern." d. "The only adverse effect of this drug is some localized stinging with application."

ANS: B Salicylate toxicity is not a problem when applied only to the face; however, if acne is extensive & the drug is applied to the trunk, back, & other locations, this could possibly occur. Monitoring for signs & symptoms of salicylism (e.g., hyperpnea, tinnitus, nausea & vomiting, & mental status changes) is indicated. Peeling & drying are adverse effects, not desired effects, of salicylic acid. Tinnitus is a symptom of systemic toxicity that is greatly concerning.

15. A patient reports taking an oral bisacodyl laxative for several years. What instructions will the provider give the patient to guide the discontinuation of the laxative? a. Stop taking the oral laxative and use a suppository until normal motility resumes. b. Stop taking the laxative immediately and expect no stool for several days. c. Switch to a bulk-forming laxative, such as methylcellulose. d. Withdraw from the laxative slowly to avoid a rebound constipation effect

ANS: B The first step in breaking the laxative habit is abrupt cessation of laxative use. Bowel movements will be absent for several days after laxative withdrawal. Using a suppository, a bulk-forming laxative, or tapering the laxative only prolongs the habit and prevents normal function from returning.

46. Which patient will the provider determine should begin treatment for tuberculosis? a. A patient with HIV and a tuberculin skin test result of a 4-mm region of induration b. A recent immigrant from a country with a high prevalence of TB with a 10-mm region of induration c. A patient with no known risk factors who has a job-related tuberculin skin test result of a 12-mm area of induration d. An intravenous drug abuser with a tuberculin skin test result of a 5-mm region of induration

ANS: B The immigrant is considered to be at moderate risk, meaning that a 10-mm area of induration on a tuberculin skin test (TST) is considered a positive result. After being evaluated for active TB, this patient should be treated for latent TB. A patient with HIV is considered high risk, but this patient has a negative TST result of less than 5 mm. For a low-risk patient receiving a screening TST for a job, the area of induration must be 15 mm or greater to be considered a positive result. An IV drug abuser is in the moderate-risk category; an area of induration of 10 mm or greater is needed to be considered a positive TST result.

76. An adolescent patient with mild cervicitis is diagnosed with gonorrhea. The provider will order which treatment regimen? a. Azithromycin, 1 g PO once, and doxycycline, 100 mg PO twice daily for 7 days b. Ceftriaxone, 250 mg IM once, & azithromycin, 1 g PO once c. Ceftriaxone, 125 mg IM once d. Doxycycline, 100 mg IV twice daily for 12 days

ANS: B The only options for treating cervical infection with gonorrhea are cefixime & ceftriaxone. Ceftriaxone is recommended over cefixime because of antibiotic resistance to cefixime. Because a high percentage of patients with gonorrhea also have chlamydial infections, they should be treated with either doxycycline or azithromycin until a chlamydial infection has been ruled out. Azithromycin combined with doxycycline would not treat gonorrhea. Ceftriaxone would treat gonorrhea only. Doxycycline would treat chlamydia only.

2. A patient with a history of peptic ulcer disease takes ranitidine and sucralfate. The patient reports that discomfort is usually controlled but that symptoms occasionally flare up. What action will the provider take to address the patient's concern? a. Order an endoscopic examination. b. Discuss testing for H. pylori and an antibiotic. c. Discuss switching to a proton pump inhibitor. d. Counsel the patient to avoid caffeine.

ANS: B The recommendation for all patients with gastric or duodenal ulcers and documented Helicobacter pylori infection is treatment with antibiotics. The provider is correct in ordering testing for this organism and to suggest adding an antibiotic to this patient's regimen. An endoscopic examination is not recommended. Changing to a proton pump inhibitor may not address the underlying cause. There is no evidence that caffeine contributes to peptic ulcer disease (PUD).

21. A patient who experiences motion sickness plans to go on a cruise. The prescriber orders transdermal scopolamine [Transderm Scop]. The patient asks why an oral agent is not ordered. What response will the provider give to explain that the benefit of a transdermal preparation? a. The patch can be applied as needed at the first sign of nausea. b. This form has less intense anticholinergic effects than the oral form. c. The patch is less sedating than the oral preparation. d. This method provides direct effects, because it is placed close to the vestibular apparatus of the ear.

ANS: B The transdermal system of scopolamine is preferred, because it may have less intense anticholinergic effects than oral or subcutaneous formulations. Anti-nausea medications for motion sickness are more effective if given prophylactically than after symptoms begin. Sedation side effects are similar with all forms. Placement near the ear does not cause the medication to absorb directly into the vestibular apparatus.

21. A patient who has been taking a glucocorticoid for several months arrives in the clinic. The patient's cheeks appear full and there is a prominent hump of fat present on the upper back. The provider will order which test? a. Liver function tests b. Serum electrolytes c. Tuberculin skin test d. Vitamin D levels

ANS: B This patient shows signs of iatrogenic Cushing syndrome, which may include serum electrolyte disturbances; therefore, the electrolyte levels should be monitored. Liver function tests, tuberculin skin testing, and vitamin D levels are not indicated.

5. An adult male patient will begin androgen therapy for testicular failure. Which statement by the patient indicates understanding of the treatment regimen as provided by the prescriber? a. "I will need to periodic x-rays." b. "My libido may improve." c. "This may decrease my hair loss." d. "This will restore fertility."

ANS: B Treatment with androgen replacement therapy in patients with testicular failure helps to restore libido. A side effect of androgens is premature epiphyseal closure; this is not a concern in adults, so radiographs to evaluate this are not indicated. Androgens do not restore fertility. Hair loss won't decrease and it may increase.

478. A nurse is teaching a patient who will begin taking methimazole [Tapazole] for Graves disease about the medication. Which statement by the patient indicates understanding of the teaching? a. "Because of the risk for liver toxicity, I will need frequent liver function tests." b. "I should report a sore throat or fever to my provider if either occurs." c. "I will need a complete blood count every few months." d. "It is safe to get pregnant while taking this medication."

ANS: B b. "I should report a sore throat or fever to my provider if either occurs." Agranulocytosis is rare but can occur with methimazole, so patients should report signs of infection, such as a sore throat or fever. Liver toxicity is not a side effect, so liver function tests are not indicated. Because agranulocytosis often develops rapidly, periodic blood counts do not guarantee early detection. Methimazole is contraindicated in the first trimester of pregnancy.

4. A provider orders thioridazine for a patient with diabetes mellitus who is diagnosed with schizophrenia. The patient requests olanzapine, which the patient has seen advertised on television. Which response will the provider give to address the primary reason the patient is not being prescribed olanzapine? a. "Olanzapine is more expensive than thioridazine." b. "Olanzapine causes more metabolic side effects than thioridazine." c. "Thioridazine has fewer side effects than olanzapine." d. "Thioridazine has a faster onset of action than olanzapine."

ANS: B b. "Olanzapine causes more metabolic side effects than thioridazine." Olanzapine is an SGA & although it has fewer extrapyramidal side effects than the FGA the provider has ordered, it has an increased risk of metabolic side effects, which is contraindicated in patients with diabetes. It is more expensive, but this is not the most important reason for not prescribing it. Thioridazine has more side effects than olanzapine, but the side effects caused by olanzapine are more critical for this patient. Thioridazine does not have a faster onset of action.

6. An infant developed a pruritic rash following exposure to an allergen. The infant's parents ask the provider about using a topical antihistamine. What information should the provider use to address the parents' question? a. Antihistamines given by this route are not absorbed as well in children. b. Applying an antihistamine to the skin can cause toxicity in this age group. c. The child will also need oral medication to achieve effective results. d. Topical medications have fewer side effects than those given by other routes.

ANS: B b. Applying an antihistamine to the skin can cause toxicity in this age group. Drug absorption through the skin is more rapid in infants, because their skin is thinner & has greater blood flow; therefore, infants are at increased risk of toxicity from topical drugs. Because of increased drug absorption through the skin, infants should not be given additional drugs via other routes. If a drug is more likely to be absorbed rapidly, it will have more side effects.

464. A nurse is educating the staff nurses about ketoacidosis. To evaluate the group's understanding, the nurse asks, "Which sign or symptom would not be consistent with ketoacidosis?" The group gives which correct answer? a. Blood glucose level of 600 mg/dL b. Blood glucose level of 60 mg/dL c. Acidosis d. Ketones in the urine

ANS: B b. Blood glucose level of 60 mg/dL A patient with diabetic ketoacidosis (DKA) has a high glucose level (at least 500 mg/dL or higher); therefore, a glucose level of 60 mg/dL would not be consistent with DKA. A blood glucose level of 600 mg/dL, acidosis, & ketones in the urine are consistent with DKA.

17. Which patient ethnic ancestry creates a risk factor that may result in minimal beneficial response to tamoxifen therapy? a. African b. French c. Native American d. Japanese

ANS: B b. French Between 8% & 10% of women of European ancestry have a gene variant that prevents the effective metabolism of tamoxifen that negatively affects the medication's therapeutic effect. None of the other options present with a similar risk factor.

481. A nurse caring for a patient notes that the patient has a temperature of 104°F & a heart rate of 110 beats/minute. The patient's skin is warm & moist, & the patient complains that the room is too warm. The patient appears nervous & has protuberant eyes. The nurse will contact the provider to discuss: a. cretinism. b. Graves disease. c. myxedema. d. Plummer disease.

ANS: B b. Graves disease. The signs & symptoms in this patient are consistent with hyperthyroidism, & because the patient's eyes are protuberant, they also are consistent with Graves disease. Cretinism is hypothyroidism in children. Myxedema is severe hypothyroidism. Plummer disease is a hyperthyroidism condition without exophthalmos.

479. A patient is admitted to the hospital & will begin taking levothyroxine [Synthroid]. The nurse learns that the patient also takes warfarin [Coumadin]. The nurse will notify the provider to discuss the dose. a. reducing; levothyroxine b. reducing; warfarin c. increasing; levothyroxine d. increasing; warfarin

ANS: B b. reducing; warfarin Levothyroxine accelerates the degradation of vitamin K-dependent clotting factors, which enhances the effects of warfarin. Patients taking warfarin who start taking levothyroxine may need to have their warfarin dose reduced. It is not correct to increase or decrease the levothyroxine dose or to increase the warfarin dose.

1. A child with allergic rhinitis has used budesonide for several years. The parents are concerned that the child's rate of growth has slowed. What action will the provider take to address the parent's concerns? a. Reassure the parents that this is an expected side effect. b. Discuss changing to fluticasone with the parents. c. Advise the parents to administer budesonide only when symptoms are severe. d. Advise the parents that antihistamines work as well as intranasal glucocorticoids.

ANS: B A worrisome systemic effect of intranasal glucocorticoids is suppression of linear growth in children. Although rare, it can occur; however, it is less likely with fluticasone and mometasone, so these two preparations are better options for children. Attempting to reassure parents that this is an expected side effect is not very reassuring. Intranasal glucocorticoids should be given daily and not as needed. Antihistamines are not as effective as glucocorticoids, because antihistamines work only against one mediator of allergic inflammation.

16. A pregnant patient in her third trimester asks the nurse whether she can take aspirin for headaches. Which response by the nurse is correct? a. "Yes, but it is safe during only the second and third trimesters of pregnancy." b. "Aspirin may cause premature closure of the ductus arteriosus in your baby so should be avoided." c. "Aspirin may induce premature labor and should be avoided in the third trimester." d. "No, you should use a first-generation nonsteroidal antiinflammatory medication."

ANS: B Aspirin poses risks to the pregnant patient and her fetus, including premature closure of the ductus arteriosus. ASA is not safe, especially in the third trimester, because it can cause anemia and can contribute to postpartum hemorrhage. ASA does not induce labor but can prolong labor by inhibiting prostaglandin synthesis. NSAIDs have similar effects and also should be avoided.

18. A 60-year-old female patient is about to begin long-term therapy with a glucocorticoid. Which intervention will the provider prescribe to help minimize the patient's risk of developing osteoporosis? a. Baseline vitamin D level b. Calcium and vitamin D supplements c. Estrogen therapy d. Skeletal x-rays before treatment

ANS: B Calcium and vitamin D supplements can help minimize the patient's risk of developing osteoporosis. A baseline vitamin D level is not recommended. Estrogen therapy can help in postmenopausal women, but its risks outweigh its benefits at this patient's age. Patients should undergo evaluation of the bone mineral density of the lower spine, not skeletal x-rays.

1. A patient with advanced renal carcinoma is prescribed aldesleukin as part of the treatment regimen. What adverse effect of this drug will be a concern for the provider? a. Hypertension b. Capillary leakage syndrome c. Hyperglycemia d. Hyperuricemia

ANS: B Capillary leakage syndrome, an adverse effect of aldesleukin, is a potentially fatal reaction characterized by hypotension & reduced organ perfusion. Aldesleukin is not associated with hypertension, hyperglycemia, or hyperuricemia.

15. A patient who is hospitalized for an acute gout attack continues to ha moderate to severe pain despite receiving several doses of hourly oral colchicine. When the patient begins vomiting after the next dose, what action will the provider take? a. Lower the dose of colchicine. b. Put a temporary hold on the colchicine. c. Explain that this is a common, but temporary, side effect. d. Prescribe an antiemetic.

ANS: B Colchicine should be discontinued immediately, regardless of the status of the joint pain, if gastrointestinal (GI) symptoms occur. The patient's symptoms indicate injury to the GI endothelium. Once damage begins to occur, lowering the dose is not indicated. Vomiting indicates GI toxicity; this is not temporary. An antiemetic may be useful for stopping the vomiting, but it doesn't address the GI toxicity that is causing the vomiting.

67. A patient diagnosed with hepatitis B begins treatment with adefovir and asks how long the drug therapy will last. What time frame will the prescriber give the patient? a. A lifetime b. An indefinite, prolonged period of time c. 48 weeks d. Until nephrotoxicity occurs

ANS: B Current guidelines recommend treatment only for patients at highest risk; it is unknown whether treatment should continue lifelong. Treatment is usually prolonged, without a specific period of time. Nephrotoxicity is common but is not the deciding factor when determining length of effective treatment.

9. A patient who has seasonal allergies works as a truck driver and needs the least sedating antihistamine to control symptoms. Which medication will the provider recommend? a. Cetirizine b. Fexofenadine c. Levocetirizine d. Loratadine

ANS: B Fexofenadine is the least sedating of the second-generation antihistamines and so is the least likely to have synergistic effects with alcohol. Cetirizine, levocetirizine, and loratadine all have sedative side effects to some extent and thus would be less safe.

1. A 6-year-old child presents with crying due to ear pain. Tympanic membranes are erythematous, bulging, & immobile, but intact. In addition to antibiotic therapy, what will the provider recommend for pain management? a. Low dose aspirin b. Lidocaine ear drops c. Prednisone d. A tympanostomy to relieve pressure in the middle ear

ANS: B For children over age 5 years, the AAP guidelines recommend topical anesthetic ear drops for pain relief if the tympanic membrane is not perforated. Tympanostomy is not indicated at this time. Aspirin is contraindicated in young children as it can contribute to the development of Reye syndrome. Glucocorticoids such as prednisone are not indicated.

14. A 14-year-old female patient has moderate acne that has not responded to topical drugs. Which treatment will the provider discuss with the patient & parents? a. Combination oral contraceptives b. Doxycycline c. Isotretinoin d. Spironolactone

ANS: B For moderate to severe acne, oral antibiotics are indicated. Doxycycline is a drug of first choice. Hormonal agents, such as oral contraceptive pills (OCPs), are used in female patients who are at least 15 years old. Isotretinoin is used for severe acne when other treatments fail. Spironolactone is used when OCPs fail.

17. A patient who was prescribed cromolyn ophthalmic drops for severe allergic conjunctivitis reports persistence of allergic symptoms after 2 days of treatment. What additional drug will the provider consider prescribing while waiting for cromolyn to reach maximal effectiveness? a. An ophthalmic demulcent b. H1-receptor antagonists c. Glucocorticoid drops d. Ocular decongestants

ANS: B Histamine receptor antagonists can be used to provide immediate symptom relief, so until the cromolyn has provided relief, they may be useful for treating symptoms. Demulcents only add moisture to the eye & do not prevent chemical mediators from causing symptoms. Glucocorticoids are used for inflammatory disorders of the eye but are not first-line agents. Ocular decongestants are used to treat redness caused by minor irritants but will not address the itching & other symptoms.

10. A patient is admitted to the hospital & is prescribed levothyroxine. Assessment data show that the patient also takes warfarin. The provider will make what medication dosage-related change? a. Reducing levothyroxine b. Reducing warfarin c. Increasing levothyroxine d. Increasing warfarin

ANS: B Levothyroxine accelerates the degradation of vitamin K dependent clotting factors, which enhances the effects of warfarin. Patients taking warfarin who start taking levothyroxine may need to have their warfarin dose reduced. It is not correct to increase or decrease the levothyroxine dose or to increase the warfarin dose.

13. A pregnant patient recently began treatment for hypothyroidism. What response will the provider give when the patient shares that she does not want to take medications while she is pregnant? a. "Hypothyroidism is a normal effect of pregnancy and usually is of no consequence." b. "Your baby will likely be born with permanent neuropsychologic deficits if the condition is not treated." c. "No danger to the fetus exists until the third trimester." d. "Treatment is required only when you are experiencing symptoms."

ANS: B Maternal hypothyroidism can result in permanent neuropsychologic deficits in the child. Hypothyroidism is not a normal effect of pregnancy & is a serious condition that can affect both mother & fetus. The greatest danger to the fetus occurs in the first trimester, because the thyroid does not fully develop until the second trimester. Early identification is essential. Symptoms often are vague. Treatment should begin as soon as possible, or intellectual disabilities & other developmental problems may occur.

12. A patient will begin initial treatment for severe acne. Which regimen will the provider order? a. Clindamycin/benzoyl peroxide and tretinoin b. Doxycycline and tretinoin c. Erythromycin and benzoyl peroxide d. Topical clindamycin and isotretinoin

ANS: B Oral antibiotics are used for moderate to severe acne & are usually combined with a topical retinoid. Combination clindamycin/benzoyl peroxide & tretinoin are used for mild to moderate acne. Erythromycin can be used as an oral antibiotic but would need to be combined with a topical retinoid. Isotretinoin is used for severe acne that has not responded to other treatments.

14. A provider has a routine order for sodium phosphate as a bowel cleanser before a patient's colonoscopy. The lab report shows that the patient's creatinine clearance and blood urea nitrogen are both elevated. What will the provider do in response to these lab results? a. Reduce the amount of fluid given with the laxative to prevent fluid retention. b. Prescribe polyethylene glycol and electrolytes (PEG-ELS) instead. c. Suggest that the patient reduce the dietary sodium intake. d. Prescribe a laxative suppository instead.

ANS: B PEG-ELS solutions provide an isosmotic solution and do not cause dehydration or electrolyte imbalance. They are safe to use in patients with renal impairment or cardiovascular disease. This patient's laboratory values suggest renal impairment. Sodium phosphate products can cause kidney damage; giving them with less fluid only increases this possibility. Reducing the dietary intake of sodium is not recommended. Suppositories are not effective bowel cleansing agents.

11. Parents report that their 5-year-old child has frequent motion sickness. Which antihistamine will the provider recommend? a. Desloratadine b. Dimenhydrinate c. Hydroxyzine d. Promethazine

ANS: B Some antihistamines, including dimenhydrinate and promethazine, are labeled for use in motion sickness. Promethazine, however, is contraindicated in children under age 2 years and should be used with caution in children older than 2 years because of the risks for severe respiratory depression. Desloratadine and hydroxyzine are not approved for motion sickness.

3. A provider considers cancer prophylaxis with tamoxifen for a 45-year-old patient with a family history of breast cancer. According to her health history, she had one child at age 35 & developed deep vein thrombosis (DVT) during the pregnancy. She has not had a hysterectomy. What information from the patient's history informs the provider's decision regarding tamoxifen? a. Because of her family risk & late childbearing, this drug is a good choice for her. b. Her history of DVT outweighs any possible benefits she may have with this drug. c. Since she has not had a hysterectomy, the risk of endometrial cancer is too great. d. When she turns 50 years old, this drug will carry fewer risks for her.

ANS: B Tamoxifen is a good choice for women between the ages of 40 and 49 years as a cancer preventive, except for women at risk for DVT. Family risk & late childbearing age are indications for using tamoxifen, but only in women without increased risk of DVT. The risk of endometrial cancer increases with age &, without the risk of DVT, would not be a contraindication in a woman of the patient's age. The patient's risks of DVT do not change with age.

7. A 3-year-old child who has asthma is in the clinic for a well-child checkup. The nurse notes that the child is up to date for the DTaP, hepatitis A, hepatitis B, and the MMR vaccines but has only had one each of the Hib, the rotavirus, and the PCV13 vaccines. Which vaccine will the provider prescribe for this child? a. Hib b. PCV13 c. PCV13 and Hib d. Rotavirus

ANS: B The PCV13 should be given to all children under the age of 2 years and to all healthy children between ages 2 and 5 years, especially those who have conditions that put them at high risk of serious pneumococcal disease. The Hib vaccine is only given up to age 15 months. The rotavirus vaccine is not given after 32 weeks of age.

63. A patient is taking oral ketoconazole for a systemic fungal infection. The medication administration record notes that the patient is also taking omeprazole for reflux disease. What instructions will the provider give the patient to maximize medication effectiveness? a. Take the omeprazole 1 hour before the ketoconazole. b. Take the omeprazole at least 2 hours after the ketoconazole. c. Restrict intake of dairy products. d. Wear sun glasses when outdoors to manage photosensitivity.

ANS: B The nurse should administer the omeprazole at least 2 hours after the ketoconazole to prevent a drug-to-drug interaction. Drugs that reduce gastric acidity should be administered no sooner than 2 hours after ingestion of ketoconazole, because they reduce absorption of the drug. There is no need to restrict dairy products or wear sun glasses because neither ketoconazole nor omeprazole have a potential to require such measures.

10. A patient prescribed amoxicillin for streptococcal pharyngitis reports new onset of a flat, itchy red rash on the chest & neck. Which action is most important? a. Provide a different prescription. b. Discontinue the medication. c. Prescribe an antihistamine cream. d. Assess for respiratory compromise.

ANS: B The priority action is to discontinue the medication to prevent worsening of the patient's symptoms. A different prescription would be provided, topical antihistamine may be administered, & the patient would be assessed for respiratory involvement, but these actions would not be performed first.

480. The nurse prepares a patient with Graves disease for radioactive iodine (131I) therapy. Which statement made by the patient best demonstrates understanding of 131I therapy? a. "I will have to isolate myself from my family for 1 week so that I do not expose them to radiation." b. "This drug will be taken up by the thyroid gland & will destroy the cells to reduce my hyperthyroidism." c. "This drug will help reduce my cold intolerance & weight gain." d. "I will need to take this drug on a daily basis for at least 1 year."

ANS: B b. "This drug will be taken up by the thyroid gland & will destroy the cells to reduce my hyperthyroidism." Iodine-131 can be used to destroy thyroid tissue in patients with hyperthyroidism; no further teaching is necessary. The patient does not need to isolate himself from others. The treatment will not reduce intolerance to cold, nor will it affect weight gain. The patient will not need the treatment daily.

18. Before initiating cetuximab therapy, the provider will order epidermal growth factor receptor (EGFR) testing for the patient having which condition? a. Breast cancer b. Colorectal cancer c. Bone cancer d. Brain cancer

ANS: B b. Colorectal cancer Cetuximab is used mainly for metastatic colorectal cancer. The medication works only against tumors that express EGRF; all other tumors are unresponsive. This makes testing in advance of treatment required. Cetuximab is not appropriate for any of the other conditions listed.

3. A patient with type 1 diabetes recently became pregnant. What blood glucose testing schedule will the provider recommended during the pregnancy? a. Before each meal and before bed b. In the morning for a fasting level and at 4:00 PM for the peak level c. Six or seven times a day d. Three times a day, along with urine glucose testing

ANS: C A pregnant patient with type 1 diabetes must have frequent blood sugar monitoring (e.g., six or seven times a day) to manage the patient's glucose levels & to ensure that no harm occurs to the fetus. Monitoring the blood sugar level before meals & at bedtime is not significant enough to provide the necessary glycemic control. Morning and 4:00 PM monitoring is not enough to provide glycemic control. Urine glucose testing is not sensitive enough to aid glycemic control, & monitoring three times a day is not enough.

20. A patient who is taking doxycycline for a serious infection reports anal itching. What action will the provider take? a. Prescribing an antihistamine to the patient's existing drug regimen b. Ordering liver function tests to test for hepatotoxicity c. Prescribing an antifungal drug to treat a superinfection d. Testing the patient for a C. difficile secondary infection

ANS: C A superinfection occurs secondary to suppression of drug-sensitive organisms. Overgrowth with fungi, especially Candida albicans, is common and may occur in the mouth, pharynx, vagina, or bowel. Anal itching is a sign of such an infection, not a sign of hepatotoxicity. Antihistamines will not treat the cause. C. difficile infection is characterized by profuse, watery diarrhea.

21. A patient is to begin taking doxycycline to treat a rickettsial infection. Which statement by the patient indicates a need for further teaching by the provider about this drug? a. "I should consult my provider before using laxatives or antacids while taking this drug." b. "I should not take a calcium supplement or consume dairy products with this drug." c. "I should take this drug with food to ensure more complete absorption." d. "If I get diarrhea, I should stop taking the drug and let my provider know immediately."

ANS: C Absorption of tetracyclines is reduced in the presence of food. The tetracyclines form insoluble chelates with calcium, iron, magnesium, aluminum, and zinc, so patients should not take tetracyclines with dairy products, calcium supplements, or drugs containing these minerals. Patients who experience diarrhea should stop taking the drug and notify the provider so they can be tested for C. difficile infection.

6. A patient is about to receive penicillin G for an infection that is highly sensitive to this drug. While obtaining the patient's medication history, the nurse learns that the patient experienced a rash when given amoxicillin as a child 20 years earlier. What action will the provider take when made aware of the patient's past reaction to amoxicillin? a. Order a cephalosporin. b. Reassure the patient that allergic responses diminish over time. c. Request an order for a skin test to assess the current risk. d. Order a desensitization schedule to be used to administer the drug safely.

ANS: C Allergy to penicillin can decrease over time; therefore, in patients with a previous allergic reaction who need to take penicillin, skin tests can be performed to assess the current risk. Until this risk is known, changing to a cephalosporin is not necessary. Reassuring the patient that allergic responses will diminish is not correct, because this is not always the case; the occurrence of a reaction must be confirmed with skin tests. Desensitizing schedules are used when patients are known to be allergic and the drug is required anyway.

5. A patient is receiving intravenous potassium penicillin G, 2 million units to be administered over 1 hour. At 1900, the nurse notes that the dose hung at 1830 has infused completely. What action will the provider take when notified of this medication error? a. Order an immediate assessment of the skin at the infusion site for signs of tissue necrosis. b. Order that the patient be closely observed for confusion and other neurotoxic effects. c. Order a serum electrolyte test and cardiac monitoring STAT. d. Request immediate notification of any bizarre behaviors demonstrated by the patient.

ANS: C Although penicillin G is the least toxic of all antibiotics, certain adverse effects may be caused by compounds coadministered with penicillin. When large doses of potassium penicillin G are administered rapidly, hyperkalemia can occur, which can cause fatal dysrhythmias. When penicillin G is administered IM, tissue necrosis occurs with inadvertent intraarterial injection. Confusion, seizures, and hallucinations can occur if blood levels of the drug are too high. Bizarre behaviors result with large IV doses of procaine penicillin G.

23. The provider is reviewing the culture results of a patient receiving an aminoglycoside. The report reveals an anaerobic organism as the cause of infection. What action will the provider take? a. Discuss an increased risk of aminoglycoside toxicity with the patient. b. Continue the aminoglycoside as ordered. c. Prescribe a different class of antibiotic. d. Add a penicillin to the patient's drug regimen.

ANS: C Aminoglycosides are not effective against anaerobic microbes, so another class of antibiotics is indicated. There is no associated increase in aminoglycoside toxicity with anaerobic infection. The aminoglycoside will not be effective, so continuing to administer this drug is not indicated. Adding another antibiotic is not useful, because the aminoglycoside is not necessary.

27. A patient has allergies & takes an antihistamine. The patient wants to know how the drug works. The nurse understands that antihistamines work because they are what? a. Activators b. Agonists c. Antagonists d. Antidotes

ANS: C Antihistamines bind to receptors to prevent activation by histamine; this makes antihistamines antagonist drugs. Antihistamines do not activate receptors. Agonist drugs activate receptors; they are not antidotes.

19. A 6-week-old infant who has not yet received immunizations develops a severe cough. While awaiting nasopharyngeal culture results, the provider will prescribe which antibiotic? a. Clindamycin b. Doxycycline c. Erythromycin ethylsuccinate d. Penicillin G

ANS: C Erythromycin is the drug of first choice for infections caused by Bordetella pertussis, the causative agent of whooping cough. Infants who have not received their first set of immunizations are at increased risk of pertussis. Clindamycin, doxycycline, and penicillin are not recommended.

8. An older adult patient with type 2 diabetes has a history of severe hypoglycemia. The patient's partner asks the provider what A1c level they should strive to achieve. What guideline will the prescriber provide? a. Between 6.5 and 7.0 b. Below 7.0 c. Below 8.0 d. Between 7.0 and 8.5

ANS: C For patients with a history of severe hypoglycemia and those with a limited life expectancy or advanced micro- and macrovascular complications, the target A1c level should be below 8.0. For most other patients with diabetes, the target is 7.0 and below.

30. A patient is receiving intravenous gentamicin. A serum drug test reveals toxic levels. The dosing is correct, & this medication has been tolerated by this patient in the past. Which could be a probable cause of the test result? a. A loading dose was not given. b. The drug was not completely dissolved in the IV solution. c. The patient is taking another medication that binds to serum albumin. d. The medication is being given at a frequency that is longer than its half-life.

ANS: C Gentamicin binds to albumin, but only weakly, & in the presence of another drug that binds to albumin, it can rise to toxic levels in blood serum. A loading dose increases the initial amount of a drug & is used to bring drug levels to the desired plateau more quickly. A drug that is not completely dissolved carries a risk of causing embolism. A drug given at a frequency longer than the drug half-life will likely be at subtherapeutic levels & not at toxic levels.

6. A child with eczema has been treated unsuccessfully with a topical glucocorticoid for a year & now has skin atrophy & hypopigmentation. What change in therapy will the provider recommend to the parents? a. Higher potency topical glucocorticoids b. Topical keratolytic agents c. Topical immunosuppressants d. Topical nonsteroidal anti-inflammatory drugs (NSAIDs)

ANS: C If topical glucocorticoids fail to treat eczema without causing skin atrophy & hypopigmentation, topical immunosuppressants may be used. Higher potency glucocorticoids will only compound the adverse effects. Topical keratolytic agents are not indicated. Topical NSAIDs are not indicated.

16. A patient with a history of chronic alcohol use has been admitted to the unit with a diagnosis of cirrhosis. Admission laboratory reports show that the patient's ammonia level is elevated at 218 µg/dL. What medication will the provider prescribe to address the lab result? a. 0.9% NS IV b. Docusate sodium [Colace] c. Lactulose d. Polyethylene glycol [MiraLax]

ANS: C Lactulose is indicated to lower ammonia levels in patients with liver disease. No information suggests that the patient needs fluid or electrolyte replacement. Docusate sodium and polyethylene glycol are not effective at lowering ammonia levels.

39. A pregnant female patient diagnosed with bacteriuria, suprapubic pain, urinary urgency and frequency, and a low-grade fever is allergic to sulfa, ciprofloxacin, and amoxicillin. The provider will prescribe which alternative medication to treat this patient? a. Cephalexin b. Fosfomycin c. Methenamine d. Nitrofurantoin

ANS: C Methenamine is an excellent second-line drug for this patient and is indicated because of the patient's multiple drug sensitivities. It is safe in pregnancy, and there is no drug resistance. Nitrofurantoin has potential harmful effects on the fetus and should not be used during pregnancy. Single-dose regimens are not recommended in pregnant women. Cephalexin can have cross-reactivity with amoxicillin.

6. A patient is diagnosed with Zollinger-Ellison syndrome. Which medication does the provider recognize as most effective for this patient's condition? a. Cimetidine b. Esomeprazole c. Omeprazole d. Sucralfate

ANS: C Omeprazole is used to treat Zollinger-Ellison syndrome. Although cimetidine can also be used, Omeprazole is more effective and therefore is preferred. Esomeprazole and sucralfate are not indicated.

47. A patient taking isoniazid for 4 months for latent tuberculosis reports bilateral tingling and numbness of the hands and feet, as well as feeling clumsy. What action will the provider take? a. Discontinue the isoniazid. b. Lower the isoniazid dose and order rifampin. c. Order pyridoxine 100 mg per day. d. Order another tuberculin skin test to monitor disease status.

ANS: C Patients sometimes develop peripheral neuropathy, characterized by paresthesias, clumsiness, and muscle aches. If these occur, they may be reversed by administering pyridoxine (vitamin B6). It is not necessary to discontinue the isoniazid. Lowering the isoniazid dose and adding rifampin is not indicated. Rechecking the TST is not indicated.

15. A patient is prescribed metronidazole for bacterial vaginosis. Which patient history finding would be most concerning? a. Recent yeast infection b. Family history of cervical cancer c. Drinks two glasses of wine every night d. Patient is currently unemployed

ANS: C Patients taking metronidazole should be educated not to drink alcohol to prevent adverse reactions. It would be concerning that the patient drinks wine daily. History of a yeast infection may indicate increased risk for recurrence with administration of an antimicrobial. A family history of cervical cancer is not related to administration of metronidazole. Unemployment can indicate lack of insurance coverage, which may limit the patient's ability to purchase medications, but is not the most concerning patient finding.

57. A patient will begin taking an immunosuppressant medication. The provider learns that a patient about to begin immunosuppressant medication has a history of frequent candidal infections. The provider will order which drug as prophylaxis? a. Fluconazole b. Ketoconazole c. Posaconazole d. Voriconazole

ANS: C Posaconazole is used as prophylaxis for invasive Aspergillus and Candida infections in immunocompromised patients. Fluconazole, ketoconazole, and voriconazole are not used prophylactically.

19. A postmenopausal patient with a family history of breast cancer develops osteoporosis. What medication will the provider prescribe? a. Estrogen estradiol b. Pamidronate c. Raloxifene d. Teriparatide

ANS: C Raloxifene is a selective estrogen receptor modulator (SERM) that has estrogenic effects in some tissues and antiestrogenic effects in others. It can preserve bone mineral density while protecting against breast and endometrial cancers. Estrogen promotes breast cancer and would not be indicated. Pamidronate and teriparatide are not protective against breast cancer.

3. A 2-year-old child presents to the clinic with otalgia & fever. The left tympanic membrane (TM) is erythematous & bulging; the right TMs is perforated & draining. The child's parent states, "This is the fifth ear infection this year. What can we do?" What action will the provider take to address the child's ear problems? a. Prescribe both ceftriaxone [Rocephin] IM with benzocaine ear drops for pain. b. Discuss beginning prophylactic antibiotic therapy with trimethoprim/sulfamethoxazole. c. Prescribe amoxicillin/clavulanate & refer the child to an otolaryngologist. d. Prescribe both high-dose amoxicillin & the influenza vaccine.

ANS: C Recurrent AOM is defined as AOM that occurs three or more times within 6 months or four or more times in a year. Giving an antibiotic, such as amoxicillin/clavulanate, is appropriate for each episode, & referral to an ENT specialist is recommended to help reduce risk. IM Rocephin can be an appropriate treatment for an episode, but benzocaine ear drops should not be used when a TM is ruptured. Prophylactic antibiotic therapy is not recommended. High-dose amoxicillin might be an appropriate treatment for an episode, but influenza vaccine is not recommended in July.

10. Which finding indicates that terazosin has been effective for a patient with benign prostatic hypertrophy (BPH)? a. Decreased prostate size b. Increased urinary frequency c. Decreased urinary hesitation d. Decreased serum PSA levels

ANS: C Terazosin is an α1-adrenergic antagonist. These medications relax the smooth muscles of the bladder neck to improve urinary symptoms experienced with BPH. They do not decrease the size of the prostate. Increased urinary frequency is a sign of worsening BPH, not improvement.

1. A 14-year-old male patient diagnosed with hypogonadism will begin testosterone patch therapy. What information will the prescriber include when teaching the family about this therapy? a. Annual x-rays of the hands and wrists are necessary to monitor epiphyseal closure. b. Gynecomastia may occur and is a common side effect. c. The patch is applied once daily to the back or abdomen. d. Use of this drug may lead to prostate cancer later in life.

ANS: C Testosterone patches are indicated for male hypogonadism. Patches are applied once daily to the upper arm, thigh, back, or abdomen. Evaluation for epiphyseal closure should be done with radiographs every 6 months, not annually. Gynecomastia is a rare, not common, side effect that usually occurs in patients taking androgens in high doses, not to bring testosterone up to normal levels. Testosterone only accelerates the growth of prostate cancer when it occurs; it does not cause it.

20. A patient receives ondansetron and dexamethasone for chemotherapy-induced nausea and vomiting (CINV). The patient states that the drugs provide relief during and immediately after chemotherapy, but that significant nausea and vomiting occur several days after each chemotherapy treatment. What action will the prescriber take to address this outcome? a. Discuss increasing the dose of ondansetron with the patient. b. Suggest giving prolonged doses of dexamethasone. c. Suggest adding aprepitant to the medication regimen. d. Discontinue the ondansetron and prescribe aprepitant.

ANS: C The current regimen of choice for patients taking highly emetogenic drugs consists of three agents: aprepitant plus dexamethasone plus a 5-HT3 antagonist, such as ondansetron. Aprepitant has a prolonged duration of action and can prevent delayed CINV as well as acute CINV. Increasing the dose of ondansetron will not help treat the delayed CINV. Glucocorticoids should be given intermittently and for short periods to avoid side effects. Changing the ondansetron to aprepitant is not recommended.

33. A patient presents to the emergency department with chills, severe flank pain, dysuria, and urinary frequency. The patient has a temperature of 102.9°F, a pulse of 92 beats/minute, respirations of 24 breaths/minute, and a blood pressure of 119/58 mm Hg. The provider suspects that the patient is showing signs and symptoms of what pathology? a. Acute cystitis b. Urinary tract infection c. Pyelonephritis d. Prostatitis

ANS: C The provider should suspect pyelonephritis. Pyelonephritis is characterized by fever, chills, severe flank pain, dysuria, urinary urgency and frequency, and pyuria and bacteriuria. Clinical manifestations of acute cystitis include dysuria, urinary urgency and frequency, suprapubic discomfort, pyuria, and bacteriuria. Urinary tract infections (UTIs) are very general and are classified by their location. These symptoms are specific to pyelonephritis. Prostatitis is manifested by high fever, chills, malaise, myalgia, localized pain, and various UTI symptoms, but not by severe flank pain.

14. An admission history on an adult patient notes that the patient has a heart rate of 62 beats/minute, a blood pressure of 105/62 mm Hg, and a temperature of 96.2°F. The patient appears pale and reports always feeling cold and tired. The provider and patient will discuss tests to rule out what possible cause of these signs symptoms? a. Cretinism b. Graves disease c. Hypothyroidism d. Plummer disease

ANS: C This patient is showing signs of hypothyroidism: a low heart rate, low temperature, pale skin, and feeling cold and tired. In adults, thyroid deficiency is called hypothyroidism. In children, thyroid deficiency is called cretinism. Graves disease and Plummer disease are conditions caused by thyroid excess.

23. A patient reports a 6-month history of daily episodes of recurrent, crampy abdominal pain followed by diarrhea that usually relieves the pain. The patient undergoes a colonoscopy, for which the findings are normal. What will the provider discuss with the patient? a. Prescribing antispasmodic medication b. Avoiding food containing lactose and gluten c. Keeping a food, stress, and symptom diary d. Prescribing antidiarrheal drugs to manage symptoms

ANS: C This patient shows signs of irritable bowel syndrome (IBS), which can be managed with drug and nondrug therapies. Patients should be taught to keep a log to identify foods and stressors that trigger symptoms. Antispasmodic medications frequently are used, but there is no clear evidence of their benefit. Patients with malabsorption disorders may need to avoid lactose or gluten but only if indicated. Antidiarrheal drugs do not have clear benefits, even though they are commonly used.

473. A patient who has type 2 diabetes will begin taking glipizide [Glucotrol]. Which statement by the patient is concerning to the nurse? a. "I will begin by taking this once daily with breakfast." b. "It is safe to drink grapefruit juice while taking this drug." c. "I may continue to have a glass of wine with dinner." d. "I will need to check my blood sugar once daily or more."

ANS: C c. "I may continue to have a glass of wine with dinner." Glipizide is a sulfonylurea antidiabetic agent & can cause a disulfiram-like reaction when combined with alcohol. Patients should be taught to avoid alcohol while taking this medication. The initial dosing is once daily with breakfast. There is no drug interaction with grapefruit juice. Patients will need to monitor their blood glucose.

471. An elderly patient who has type 2 diabetes has a history of severe hypoglycemia. The patient's spouse asks the nurse what the optimal A1c level is for the patient. Which is correct? a. Between 6.5 and 7.0 b. Below 7.0 c. Below 8.0 d. Between 7.0 and 8.5

ANS: C c. Below 8.0 For patients with a history of severe hypoglycemia & those with a limited life expectancy or advanced micro- & macrovascular complications, the target A1c level should be below 8.0. For most other patients with diabetes, the target is 7.0 and below.

476. A patient with hypothyroidism begins taking PO levothyroxine [Synthroid]. The nurse assesses the patient at the beginning of the shift & notes a heart rate of 62 beats/minute and a temperature of 97.2°F. The patient is lethargic & difficult to arouse. The nurse will contact the provider to request an order for which drug? a. Beta blocker b. Increased dose of PO levothyroxine c. Intravenous levothyroxine d. Methimazole [Tapazole]

ANS: C c. Intravenous levothyroxine Intravenous administration of levothyroxine is used for myxedema coma. This patient is showing signs of severe hypothyroidism, or myxedema. A beta blocker is useful in patients who show signs of hyperthyroidism to minimize cardiac effects. Because the half-life of oral levothyroxine is so long, increasing the PO dose will not provide immediate relief of this patient's symptoms. Methimazole is used to treat hyperthyroidism.

475. Insulin glargine is prescribed for a hospitalized patient who has diabetes. When will the nurse expect to administer this drug? a. Approximately 15 to 30 minutes before each meal b. In the morning and at 4:00 PM c. Once daily at bedtime d. After meals & at bedtime

ANS: C c. Once daily at bedtime Glargine insulin is indicated for once-daily subcutaneous administration to treat adults & children with type 1 diabetes & adults with type 2 diabetes. According to the package labeling, the once-daily injection should be given at bedtime. Glargine insulin should not be given more than once a day, although some patients require bid dosing to achieve a full 24 hours of basal coverage.

462. A patient with type 1 diabetes recently became pregnant. The nurse plans a blood glucose testing schedule for her. What is the recommended monitoring schedule? a. Before each meal and before bed. b. In the morning for a fasting level and at 4:00 PM for the peak level. c. Six or seven times a day. d. Three times a day, along with urine glucose testing.

ANS: C c. Six or seven times a day. A pregnant patient with type 1 diabetes must have frequent blood sugar monitoring (e.g., six or seven times a day) to manage both the patient & the fetus so that no teratogenic effects occur. Monitoring the blood sugar level before meals & at bedtime is not significant enough to provide the necessary glycemic control. Morning & 4:00 PM monitoring is not enough to provide glycemic control. Urine glucose testing is not sensitive enough to aid glycemic control, & monitoring three times a day is not enough.

5. A patient receiving intravenous gentamicin has a toxic serum drug level. The prescriber confirms that the dosing is correct. Which possible cause of this situation will the provide explore? a. Whether a loading dose was administered b. If the drug was completely dissolved in the IV solution c. Whether patient is taking a medication that binds to serum albumin d. If the ordered dose frequency is longer than the gentamicin half-life

ANS: C c. Whether patient is taking a medication that binds to serum albumin Gentamicin binds to albumin, but only weakly, & in the presence of another drug that binds to albumin, it can rise to toxic levels in blood serum. A loading dose increases the initial amount of a drug & is used to bring drug levels to the desired plateau more quickly. A drug that is not completely dissolved carries a risk of causing embolism but this addresses a different concern. A drug given at a frequency longer than the drug half-life will likely be at subtherapeutic levels & not at toxic levels.

11. A female patient with type 1 diabetes and obesity is prescribed lorcaserin to help with weight loss. What statement will the provide include when teaching this patient about lorcaserin? a. "This drug is safe to take even if you become pregnant." b. "This drug does not have serious adverse effects." c. "You will need to check your blood glucose levels more often." d. "You will need to increase your insulin dose to prevent hyperglycemia."

ANS: C About 30% of patients with diabetes will experience an increase in hypoglycemic episodes when taking lorcaserin. It should not be taken by women who are pregnant. It has uncommon, but severe adverse effects. Since it can cause hypoglycemia, patients should not increase insulin doses.

13. A patient has severe acne that has been refractory to treatment with tetracycline, topical tretinoin, & benzoyl peroxide. The provider plans to begin treatment with isotretinoin. The provider will include which statement when teaching this patient about this drug? a. "Alcohol may be consumed in moderation when taking this drug." b. "Skin rash and excessive body hair growth are common with this drug." c. "Tetracycline must be discontinued before beginning the isotretinoin." d. "Two pregnancy tests are required before each monthly refill of your prescription."

ANS: C Adverse effects of isotretinoin can be increased by tetracycline, so tetracycline must be discontinued before therapy is started. Alcohol should be avoided, since it can potentiate hypertriglyceridemia. Skin rash & hair loss can occur. Two pregnancy tests are required at the beginning of therapy; at each refill, only one pregnancy test is required.

7. A patient with severe alcohol use disorder is brought to the emergency department with ataxia, double-vision, and confusion. The examination reveals nystagmus. Which vitamin will the provider prescribe? a. Ascorbic acid IV b. Intramuscular pyridoxine c. Intravenous thiamine d. Nicotinic acid PO

ANS: C Alcoholics who are malnourished commonly have a thiamine deficiency. When severe, this can cause a neurological disorder called Wernicke-Korsakoff syndrome, which is characterized by nystagmus, diplopia, ataxia, confusion, and short-term memory loss. Parenteral thiamine is indicated for treatment. Ascorbic acid is given to treat vitamin C deficiency, or scurvy. Pyridoxine is given for vitamin B6 deficiency, also common in alcoholics, but is characterized by seborrheic dermatitis and peripheral neuropathy. Nicotinic acid is used for niacin deficiency, which is characterized by severe dry, rough skin.

69. The nurse is caring for a patient who is taking a protease inhibitor (PI). Upon review of the laboratory test results, the nurse notes that the patient has newly elevated plasma triglycerides and cholesterol. Upon being notified of the test results, what action will the provider take? a. Prescribe lovastatin. b. Prescribe simvastatin. c. Discuss diet modification and exercise. d. Prescribe pancrease.

ANS: C All PIs can elevate plasma levels of cholesterol and triglycerides. Potential interventions for hyperlipidemia include modified diet, exercise, and lipid-lowering agents. Lovastatin and simvastatin should be avoided, because they can accumulate to dangerous levels. Pancreas is not indicated to lower triglycerides and cholesterol.

12. A patient who takes aspirin daily is scheduled for surgery in 1 week. What action will the provider suggest to minimize the patient's risk for injury? a. Continue to use aspirin as scheduled. b. Reduce the aspirin dosage by half until after surgery. c. Stop taking aspirin immediately. d. Stop taking aspirin 3 days before surgery.

ANS: C Aspirin should be withdrawn at least 1 week before surgery. Aspirin cannot be continued as scheduled, because the risk for bleeding is too great. An interval of 3 days is not long enough for the bleeding effects of aspirin to be reversed. Cutting the dose in half would not reduce the effects of bleeding associated with aspirin use.

63. A patient with type 1 diabetes is taking NPH insulin, 30 units every day. A nurse notes that the patient is also taking metoprolol [Lopressor]. What education should the nurse provide to the patient? a. "Metoprolol has no effect on diabetes mellitus or on your insulin requirements." b. "Metoprolol interferes with the effects of insulin, so you may need to increase your insulin dose." c. "Metoprolol may mask signs of hypoglycemia, so you need to monitor your blood glucose closely." d. "Metoprolol may potentiate the effects of the insulin, so the dose should be reduced."

ANS: C Because metoprolol may mask the signs of hypoglycemia, the patient should monitor the blood glucose closely & report changes to the prescriber. Metoprolol does have an indirect effect on diabetes mellitus &/or insulin requirements in that it may mask the signs of hypoglycemia, causing the patient to make a healthcare decision based on the drug-to-drug interaction rather than actual physiologic factors. The patient should not increase the insulin, because metoprolol will cause a decrease in blood glucose, increasing the risk of a hypoglycemic reaction. The patient should not reduce the dose of insulin when taking metoprolol, because this might alter serum glucose levels.

15. A patient diagnosed with C. difficile infection is considered a high risk for recurrence. Which medication will the provider prescribed? a. Dupilumab b. Omalizumab c. Bezlotoxumab d. Mepolizumab

ANS: C Bezlotoxumab is used in patients currently undergoing antimicrobial treatment for C. difficile infection who are at high risk for recurrence. The other options are approved for the treatment of various types of asthma.

22. A patient recently began receiving clindamycin to treat an infection. After 8 days of treatment, the patient reports having 10 to 15 watery stools per day. What action will the provider take to address the patient's diarrhea? a. Increase the clindamycin dose to aggressively treat this infection. b. Assure the patient this is known side effect of clindamycin, and suggest consuming extra fluids. c. Discontinue the clindamycin immediately. d. Prescribe Lomotil or a bulk laxative to minimize the diarrheal symptoms.

ANS: C Clostridium difficile-associated diarrhea (CDAD) is the most severe toxicity of clindamycin; if severe diarrhea occurs the patient should be told to stop taking clindamycin immediately and to contact the provider so that treatment with vancomycin or metronidazole can be initiated. Increasing the dose of clindamycin will not treat this infection. Consuming extra fluids while still taking the clindamycin is not correct, because CDAD can be fatal if not treated. Taking Lomotil or bulk laxatives only slows the transit of the stools and does not treat the cause.

6. Insulin glargine is prescribed by the provider for a hospitalized patient with type 1 diabetes. When will the provider order this medication to be administered? a. Approximately 15 to 30 minutes before each meal b. In the morning and at 4:00 PM c. Once daily at bedtime d. After meals and at bedtime

ANS: C Glargine insulin is indicated for once-daily subcutaneous administration to treat adults and children with type 1 diabetes and adults with type 2 diabetes. According to the package labeling, the once-daily injection should be given at bedtime. Glargine insulin should not be given more than once a day, although some patients require bid dosing to achieve a full 24 hours of basal coverage.

10. A provider orders hydroxyzine for a patient with acute urticaria. What information will the provider include when teaching the patient about this drug? a. The drug will reduce redness and itching but not edema. b. This antihistamine is not likely to cause sedation. c. Alcohol should be avoided while taking the drug. d. Shortness of breath may occur while taking the drug.

ANS: C Hydroxyzine is a first-generation antihistamine and has sedative effects, so patients should be cautioned not to consume alcohol while taking the drug. In capillary beds, antihistamines reduce edema, itching, and redness. This antihistamine causes sedation. It is not associated with respiratory depression or shortness of breath at therapeutic doses.

61. A patient diagnosed with histoplasmosis is being treated with itraconazole. The provider will teach this patient to report which symptoms? a. Gynecomastia and decreased libido b. Headache and rash c. Nausea, vomiting, and anorexia d. Visual disturbances

ANS: C Itraconazole is associated with rare cases of liver failure, some of which were fatal. Patients should be instructed to report signs of liver toxicity, including nausea, vomiting, and anorexia. Ketoconazole is associated with gynecomastia and libido changes. Headache and rash are associated with fluconazole. Visual disturbances may occur with voriconazole.

8. A patient with lower abdominal pain and nausea is admitted to the hospital. The patient's abdomen is distended and firm with hypoactive bowel sounds. The patient reports of not having a bowel movement for 3 days. What initial action will the provider take? a. Order a bulk-forming laxative. b. Order a saline enema. c. Order diagnostic tests. d. Prescribe a cathartic laxative.

ANS: C Laxatives are contraindicated for patients with abdominal pain, nausea, or other symptoms of abdominal disease or an acute surgical abdomen. The patient should remain NPO and no interventions undertaken until diagnostic tests identify the cause of the patient's symptoms.

15. A patient who has been taking linezolid for 6 months develops vision problems and is worried about blindness. What response will the provider give to address the patient's concern? a. Reassure the patient that this is a harmless side effect of this drug. b. Tell the patient that blindness rarely occurs with this drug c. Tell the patient that this symptom is reversible when the drug is discontinued. d. Suggest the patient take tyramine supplements to minimize this effect.

ANS: C Linezolid is associated with neuropathy, including optic neuropathy. This is a reversible effect that will stop when the drug is withdrawn. Reassuring the patient that this is a harmless side effect is not correct. It is not an indication that blindness will occur. Tyramine supplements are not indicated.

75. A patient is diagnosed with pelvic inflammatory disease (PID). Which treatment regimen is most appropriate for reducing the risk of sterility in this patient? a. Azithromycin, 1 g PO once, & cefoxitin, 2 g IM once in the clinic b. Ceftriaxone, 250 mg IM once, with doxycycline, 100 mg PO twice daily for 14 days as an outpatient c. Doxycycline, 100 mg IV twice daily, & cefoxitin, 2 g IV every 6 hours in the hospital d. Doxycycline, 100 mg PO twice daily for 14 days, & metronidazole, 500 mg PO twice daily for 14 days in the hospital

ANS: C Many experts recommend that all patients with PID receive IV antibiotics in the hospital to minimize the risk of sterility & other complications. Medications used intravenously can be cefoxitin or cefotetan combined with doxycycline; when symptoms resolve, IV therapy may be discontinued but must be followed by PO doxycycline. Outpatient regimens are not recommended. Oral medications are not recommended for initial treatment.

27. A patient shows signs and symptoms of conjunctivitis. Which aminoglycoside will the provider order? a. Amikacin b. Kanamycin c. Neomycin d. Paromomycin

ANS: C Neomycin is used for topical treatment of infections of the eye, ear, and skin. Amikacin, kanamycin, and paromomycin are not topical treatments and are not indicated for eye infections.

25. A patient with Crohn disease will receive an initial infusion of infliximab. The prescriber explains how this drug works to treat this disease. Which statement by the patient indicates a need for further teaching? a. "I may have an increased risk of infections when taking infliximab." b. "I should report chills, fever, itching, and shortness of breath while receiving the infusion." c. "This drug sometimes provides a complete cure of inflammatory bowel disease." d. "I will take the second dose in 2 weeks, the third dose in 6 weeks, and then a dose every 8 weeks thereafter."

ANS: C None of the drugs used to treat Crohn disease are curative. Patients taking immunomodulators, such as infliximab, have an increased risk of infection. Infusion reactions may occur and include chills, fever, itching, and shortness of breath. The induction regimen is 5 mg/kg infused at 0, 2, and 6 weeks, followed by a maintenance regimen every 8 weeks.

19. A patient is prescribed ondansetron 30 minutes before initiation of chemotherapy. The patient reports that the ondansetron did not work as well as it had previously. What action will the prescriber take? a. Order that the ondansetron be given at the same time as the chemotherapy is administered. b. Prescribe a high dose of intravenous dolasetron. c. Prescribe dexamethasone to be given with the ondansetron. d. Prescribe loperamide to be given with the ondansetron.

ANS: C Ondansetron is a serotonin receptor antagonist; drugs in this class are the most effective drugs available for suppressing nausea and vomiting associated with anticancer drugs. The drug is even more effective when combined with dexamethasone. For best effect, ondansetron should be given 30 minutes before beginning chemotherapy. Dolasetron is similar to ondansetron, but when given intravenously in high doses, it is associated with fatal dysrhythmias. Loperamide is used to treat diarrhea.

1. A patient takes nonsteroidal antiinflammatory drugs (NSAIDs) for arthritis. What can the provider prescribe to prevent ulcers? a. Antibiotics b. Histamine-2 receptor antagonists c. Proton pump inhibitors d. Mucosal protectants

ANS: C Patients taking NSAIDs can use proton pump inhibitors for ulcer prophylaxis. The other agents are not used for prophylaxis.

7. A patient with an infection caused by Pseudomonas aeruginosa is being treated with piperacillin. The nurse providing care reviews the patient's laboratory reports and notes that the patient's blood urea nitrogen and serum creatinine levels are elevated. What action will the provider take when notified of the elevated lab results? a. Prescribes an aminoglycoside b. Discontinues the piperacillin and orders penicillin G c. Reduces the dosage of piperacillin d. Discontinues the piperacillin and prescribes nafcillin

ANS: C Patients with renal impairment should receive lower doses of piperacillin than patients with normal renal function. Aminoglycosides are nephrotoxic. Penicillin G and nafcillin are not effective against Pseudomonas infections.

9. A patient diagnosed with renal disease is scheduled for a colonoscopy. What drug will the provider prescribe for administration before the procedure? a. A glycerin suppository b. Magnesium hydroxide c. Polyethylene glycol plus electrolytes (PEG-ELS) d. Sodium phosphate

ANS: C Polyethylene glycol (PEG) plus electrolytes (ELS) is one of two bowel cleansers used before colonoscopy to clear the bowel. PEG-ELS products are preferred, because unlike sodium phosphate, they are isotonic and do not increase the likelihood of dehydration and electrolyte imbalance. Glycerin suppositories and magnesium hydroxide are not used for bowel cleansing.

16. Azithromycin is prescribed for a patient who develops an infection. The patient's only other medication is simvastatin. Which patient symptom will create the greatest concern for the provider? a. Nausea b. Tiredness c. Muscle pain d. Headache

ANS: C Statins can injure muscle tissue, causing muscle aches and pain known as myopathy/rhabdomyolysis. Azithromycin also can cause myopathy and therefore should be used with caution in patients concurrently taking simvastatin. Nausea, tiredness, and headache would not cause the provider as much concern as the likelihood of myopathy.

2. A prescriber is teaching a male adult patient about the use of testosterone gel. Which statement by the patient indicates an understanding of the teaching? a. "I should apply this to my forearms and neck after showering." b. "I should keep treated areas exposed to the air so that they can dry." c. "I should wash the application site before being intimate with my partner." d. "I should not swim or bathe for 3 to 4 hours after applying the gel."

ANS: C Testosterone administered via gels can be transferred to others by skin-to-skin contact; gel users should wash the application site before skin-to-skin contact with another person. The gel should be applied to clean, dry skin on the upper arms, shoulders, or abdomen and should be covered with clothing. Swimming and bathing are allowed 5 to 6 hours after application.

5. A 1-year-old child receives the MMR vaccine. The next day, the child's parent calls to report that the child has a temperature of 102.8°F. What action will the provider take? a. File an adverse event report with the Vaccine Adverse Event Reporting System (VAERS). b. Order a lab test to rule out thrombocytopenia. c. Reassure the parent that fever can occur with the MMR vaccine. d. Advise the parent to take the child to the emergency department.

ANS: C The MMR vaccine can have several adverse effects, including fever up to 103°F. This is not considered a serious effect and does not warrant filing an adverse event report with VAERS. Thrombocytopenia is a rare but serious side effect of the MMR vaccine that would take longer than 24 hours to develop and is not associated with fever. There is no need to have the parent take the child to the emergency department.

11. A patient who takes oral levothyroxine for hypothyroidism is admitted to the hospital. After the provider determines the patient has myxedema, what action will the provider take? a. Prescribe a β blocker. b. Increase the dose of levothyroxine. c. Change to intravenous levothyroxine. d. Prescribe methimazole.

ANS: C This patient is showing signs of severe hypothyroidism or myxedema. Intravenous administration of levothyroxine is used for myxedema coma. A β blocker is useful in patients who show signs of hyperthyroidism to minimize cardiac effects. Because the half-life of oral levothyroxine is so long, increasing the PO dose will not provide immediate relief of this patient's symptoms. Methimazole is used to treat hyperthyroidism.

32. A patient who takes an ACE inhibitor and an angiotensin receptor blocker (ARB) medication will begin taking TMP/SMZ to treat a urinary tract infection. Which serum electrolyte will the provider monitor closely? a. Calcium b. Chloride c. Potassium d. Sodium

ANS: C Trimethoprim suppresses renal excretion of potassium, increasing the risk of hyperkalemia. Patients at greatest risk are those taking high doses of trimethoprim and those taking other drugs that elevate potassium, including ACE inhibitors and ARB medications. Trimethoprim does not affect other serum electrolytes.

6. A provider teaches a patient about a vesicant chemotherapeutic agent. Which statement by the patient indicates a need for further teaching about this type of drug? a. "Leakage of this type of drug around the IV site may result in the need for skin grafts." b. "If the medicine in the IV line leaks around the IV site, it must be discontinued immediately." c. "These drugs may be administered orally as well as intravenously." d. "This type of drug may not be infused at a site of radiation therapy."

ANS: C Vesicants are given intravenously. Extravasation may cause severe local tissue injury, requiring skin grafts. Vesicants that infiltrate must be stopped immediately. Sites of previous irradiation should not be used.

28. A patient is receiving tobramycin three times daily. The provider has ordered a trough level with the 8:00 AM dose and will expect the level to be drawn at what time? a. 4:00 AM b. 7:00 AM c. 7:45 AM d. 8:45 AM

ANS: C When a patient is receiving divided doses of an aminoglycoside, the trough level should be drawn just before the next dose; therefore, 7:45 AM would be the appropriate time. It would not be appropriate to draw a trough at the other times listed.

MULTIPLE RESPONSE 1. An 11-year-old boy received all childhood immunizations before attending kindergarten. Which vaccines will the provider recommended for this child at his current age? (Select all that apply.) a. Hepatitis B b. PCV-23 c. Tdap d. MCV4 e. HPV

ANS: C, D, E At age 11, both males and females should receive a booster of diphtheria, tetanus, and pertussis (Tdap); the Menactra vaccine against meningitis (MCV4); and the human papillomavirus (HPV) vaccine. The PCV-23 vaccine is indicated only in high-risk patients. The MMR is not given at this age. The hepatitis B vaccine is not given at this age.

MULTIPLE RESPONSE 1. A 14-year-old male patient is admitted to the hospital following a football injury. The patient is muscular, has a deep voice, and full beard. Which tests will the provider order in response to these assessment findings? (Select all that apply.) a. Thyroid-stimulating hormone (TSH) level b. Complete blood count (CBC) with differential c. Liver function tests d. Serum glucose and hemoglobin A1c e. Serum cholesterol

ANS: C, E Adverse effects of androgen abuse can cause hepatotoxicity and an elevated serum cholesterol level, with a decrease in HDL cholesterol and an increase in LDL cholesterol. A TSH level is not necessary. A CBC is not indicated. Serum glucose and hemoglobin A1c levels are not indicated.

3. A patient is prescribed metronidazole for bacterial vaginosis. Which patient history finding would be most concerning to the provider? a. The patient had a recent yeast infection. b. There is a family history of cervical cancer. c. The patient drinks two glasses of wine every night. d. The patient is unemployed.

ANS: C. The patient drinks two glasses of wine every night. Patients taking metronidazole should be educated not to drink alcohol to prevent a disulfiram-like reaction. It would be concerning that the patient drinks wine daily. History of a yeast infection may indicate increased risk for recurrence with administration of an antimicrobial. A family history of cervical cancer is not related to administration of metronidazole. Unemployment can indicate lack of insurance coverage, which may limit the patient's ability to purchase medications; however, generic metronidazole is one of the less expensive medications.

15. A patient has a free T4 level of 0.6 ng/dL and a free T3 level of 220 pg/dL. When asked by the patient what these laboratory values mean, how will the provider respond? a. "These laboratory values indicate that you may have Graves disease." b. "These results suggest you may have hyperthyroidism." c. "We will need to obtain a total T4 and a total T3 to tell for sure." d. "We will need to obtain a TSH level to better evaluate your diagnosis."

ANS: D A free T4 level of less than 0.9 ng/dL & a free T3 level of less than 230 pg/dL are consistent with hypothyroidism, but measurement of the thyroid-stimulating hormone (TSH) level is necessary to distinguish primary hypothyroidism from secondary hypothyroidism. Total T3 & T4 levels are not as helpful as free T3 & T4 levels. These laboratory values indicate hypothyroidism, not hyperthyroid conditions such as Graves disease.

13. A patient being treated for erectile dysfunction is prescribed sildenafil 50 mg PRN. Which assessment finding would be most concerning to the provider? a. Current blood pressure reading of 118/76 b. Patient notes erection lasting at least 3 hours c. Patient reports onset of blurred vision d. New onset of snoring that wakens the patient's partner

ANS: D All the assessment findings are indicative of potential side effects of sildenafil. However, snoring that has worsened indicates intensification of obstructive sleep apnea, which can lead to airway obstruction. This patient report would be most concerning.

11. A patient being treated with warfarin to prevent thrombus develops hyperuricemia, and the provider orders allopurinol. The provider will discuss ____ the ____ dose with the patient. a. increasing; allopurinol b. increasing; warfarin c. reducing; allopurinol d. reducing; warfarin

ANS: D Allopurinol can inhibit hepatic drug-metabolizing enzymes and thus delay the metabolism of warfarin; therefore, the warfarin dose should be reduced when allopurinol is added. It is not correct to increase the allopurinol dose, increase the warfarin dose, or reduce the allopurinol dose.

8. A patient diagnosed with cystic fibrosis has a Pseudomonas aeruginosa infection and the provider has ordered aztreonam. What instruction will the provider give the patient regarding the administration of this drug? a. Take one pill twice daily. b. Take a daily dose for 28 days. c. Inhale the powdered drug as ordered three times each day. d. Use the nebulizer to administer the drug three times daily.

ANS: D Cayston is a form of aztreonam formulated for inhalation administration for patients with cystic fibrosis who have P. aeruginosa lung infections. The reconstituted powder is given using a nebulizer system three times daily for 28 days followed by 28 days off. This form of the drug is not given IM. The dose is three times daily. The drug is reconstituted and administered via a nebulizer.

17. A provider is about to prescribe prednisone to a patient for tendonitis. What item in the patient's medical history would cause the provider to reconsider that action? a. Allergic rhinitis b. Gouty arthritis c. Seborrheic dermatitis d. Systemic fungal infection

ANS: D Glucocorticoids are contraindicated in patients with a history of systemic fungal infections. Glucocorticoids are used to treat, allergic rhinitis, gout, and seborrheic dermatitis.

9. A patient prescribed dutasteride 2 weeks ago reports continued urinary hesitancy. Which action by a provider is most appropriate? a. Obtaining a urine sample b. Doubling the daily dose c. Performing a prostate examination d. Documenting these findings

ANS: D It may take up to 1 month for the effects of dutasteride to take effect. The patient should be educated about the time taken for positive effects to be achieved. A urine sample would not be necessary. Doubling the dose is not appropriate because the medication has not had sufficient time to take effect. A prostate examination may be performed, but would not reveal findings related to the use of dutasteride (and was likely done as part of the initial evaluation prior to prescribing the drug).

7. A patient with gastroesophageal reflux disease (GERD) receives a prescription for a proton pump inhibitor (PPI). What information will the provider include when teaching the patient about this drug? a. "The FDA has determined that there is a gastric cancer risk with this drug." b. "This drug will be given on a short-term basis only." c. "You may experience high magnesium levels when taking this drug." d. "You should report any fever and cough to your provider."

ANS: D PPIs can increase the risk of community-acquired and hospital-acquired pneumonia in the first few days of use and patients should be taught to report symptoms to their provider. The FDA has concluded that there is no increased risk of gastric cancer associated with PPIs. PPIs are often used long-term to treat GERD, since the risk of relapse is greater than 80%. There is a risk of hypomagnesemia, not hypermagnesemia, with long-term use.

23. The nurse is preparing to administer penicillin G intramuscularly to a child. The child's parents ask why the drug cannot be given in an oral liquid form. What is the nurse's reply? a. "This drug causes severe gastric upset if given orally." b. "This drug has a narrow therapeutic range, and the dose must be tightly controlled." c. "This drug is absorbed much too quickly in an oral form." d. "This drug would be inactivated by enzymes in the stomach."

ANS: D Penicillin G is inactivated by digestive enzymes in the stomach & cannot be given orally. It does not have a narrow therapeutic range.

9. A patient arrives in the emergency department with a heart rate of 128 beats/minute & a temperature of 105°F. The patient's skin feels hot & moist. The free T4 level is 4 ng/dL, the free T3 level is 685 pg/dL, & the TSH level is 0.1 microunits/mL. The provider caring for this patient will give what intervention priority? a. Intravenous levothyroxine b. Iodine-131 (131I) c. Methimazole d. Propylthiouracil (PTU)

ANS: D Propylthiouracil is used for patients experiencing thyroid storm, & this patient is showing signs of that condition. Levothyroxine is given IV for hypothyroidism. 131I is used in patients over 30 years of age who have not responded to other therapies for hyperthyroidism. Methimazole is used long term to treat hyperthyroidism, but PTU is more useful for emergency treatment.

66. A female patient who has hepatitis C is being treated with pegylated interferon α and ribavirin. It will be important for the provider to discuss what topic with the patient? a. If she gets pregnant, she should use the inhaled form of ribavirin. b. If she is taking oral contraceptives, she should also take a protease inhibitor. c. She should use a hormonal contraceptive to avoid pregnancy. d. She will need a monthly pregnancy test during her treatment.

ANS: D Ribavirin causes severe fetal injury and is contraindicated during pregnancy. Women taking ribavirin must rule out pregnancy before starting the drug, monthly during treatment, and monthly for 6 months after stopping treatment. Inhaled ribavirin is also embryo lethal and teratogenic. Adding a protease inhibitor will reduce the efficacy of oral contraceptives. Women using ribavirin should use two reliable forms of birth control.

31. A drug history from a patient about to receive sulfadiazine identifies that the patient takes warfarin, glipizide, and a thiazide diuretic. Based on this information, what action will the provider take? a. Change the antibiotic to TMP/SMZ. b. Increase the dose of the glipizide. c. Order daily lab testing of the patient's electrolytes. d. Order daily coagulation levels to be drawn.

ANS: D Sulfonamides interact with several drugs and through metabolism-related interactions can intensify the effects of warfarin. Patients taking both should be monitored closely for bleeding tendencies. Changing to the combination product will not help, because sulfonamides are still present. Sulfonamides intensify glipizide levels, so this drug may actually need to be reduced. Trimethoprim, not sulfonamides, raises potassium levels.

17. A patient who has been taking senna for several days notes that the urine is yellowish brown. What response will the prescriber provide when the patient expresses concern about this change? a. It indicates that renal failure has occurred. b. It is caused by dehydration, which is a laxative side effect. c. It is a sign of toxicity; this drug must be discontinued. d. It is an expected, harmless effect of senna.

ANS: D Systemic absorption of senna, followed by renal excretion, may impart a harmless yellowish brown or pink color to the urine. This symptom is not an indication of renal failure, dehydration, or toxicity.

4. A nurse caring for a 5-year-old child notes that the child has discoloration of several teeth. When taking a medication history, the nurse will ask about which group of medications? a. Glucocorticoids b. Salicylates c. Sulfonamides d. Tetracyclines

ANS: D Tetracyclines cause discoloration in developing teeth in children. Glucocorticoids are associated with growth suppression. Salicylates are associated with Reye syndrome. Sulfonamides are associated with kernicterus in newborns.

5. What method will the provider consider the most reliable measure for assessing a patient's diabetes control over the preceding 3-month period? a. Self-monitoring blood glucose (SMBG) graph report b. Random blood glucose level c. Fasting blood glucose level d. Glycosylated hemoglobin level (A1c)

ANS: D The glycosylated hemoglobin level tells much about what the plasma glucose concentration has been, on average, over the previous 2 to 3 months. The SMBG graph report is done by the patient and indicates each blood sugar level the patient has on a daily basis. It is not as reliable as the glycosylated hemoglobin level, because the equipment used might not be accurate and the testing may not reflect actual measurements 100% of the time. Random blood sugar levels are not as accurate as the glycosylated hemoglobin level for the same reason that the SMBG is not. One fasting blood glucose level indicates the patient's blood sugar level for that one time when it was obtained but is not reflective of a 3-month period.

43. A patient is beginning treatment for active tuberculosis (TB) in a region with little drug-resistant TB. Which treatment regimen will the provider prescribe initially? a. Isoniazid and pyrazinamide b. Isoniazid, pyrazinamide, and ethambutol c. Rifampin, pyrazinamide, and ethambutol d. Isoniazid, rifampin, pyrazinamide, and ethambutol

ANS: D The induction phase of treatment for patients in a region without drug resistance is the same as for patients who are human immunodeficiency virus (HIV) negative or HIV positive and includes isoniazid, rifampin, pyrazinamide, and ethambutol. It is not correct to begin with two drugs. The three-drug regimen is used for inductions in areas with resistance to either isoniazid or rifampin.

73. A patient is HIV positive & the provider is about to prescribe zidovudine. Before the medication therapy is initiated the provider will review which of the patient's latest laboratory values? a. Ketones in the urine & blood b. Serum immunoglobulin levels c. Serum lactate dehydrogenase d. Complete blood count (CBC)

ANS: D The patient's CBC should be reviewed to determine whether the patient has anemia & neutropenia. Ketones are not an adverse effect of zidovudine. Nothing indicates a need to monitor the immunoglobulin levels or serum lactate dehydrogenase.

34. An older male patient comes to the clinic with reports of chills, malaise, myalgia, localized pain, dysuria, nocturia, and urinary retention. The provider will suspect what cause for the patient's symptomology? a. Acute cystitis b. Urinary tract infection c. Pyelonephritis d. Prostatitis

ANS: D The provider should suspect prostatitis, which is manifested by high fever, chills, malaise, myalgia, and localized pain, and may also be manifested by dysuria, nocturia, and urinary urgency, frequency, and retention. Clinical manifestations of acute cystitis include dysuria, urinary urgency and frequency, suprapubic discomfort, pyuria, and bacteriuria. Urinary tract infections are very general and are classified by their location. Pyelonephritis is characterized by fever, chills, severe flank pain, dysuria, and urinary frequency and urgency, as well as by pyuria and bacteriuria.

1. A provider caring for a patient infected with the human immunodeficiency virus (HIV) will be most concerned about which lab result? a. High level of eosinophils b. Low neutrophil count c. Decreased red blood cell (RBC) count d. Very low helper T lymphocyte count

ANS: D The provider understands that this patient may be vulnerable to opportunistic infections, especially if there were an indication of the HIV conversion to acquired immunodeficiency syndrome (AIDS). A very low helper T lymphocyte count would most concern the nurse, because the helper T cells are essential to the immune system, and people with AIDS have a low or deficient count. A high level of eosinophils likely indicates an allergy. A decreased RBC count is unrelated to HIV infection and is only a concern if it is low. A high neutrophil count, not a low count, indicates infection. Low counts often reflect an increased percentage of another WBC in the differential.

13. A patient has had three gouty flare-ups in the past year. Which type of drug will the provider prescribe to decrease the frequency of flare-ups? a. Antigout antiinflammatory drug b. Glucocorticoid c. Nonsteroidal antiinflammatory drug d. Urate-lowering drug

ANS: D The provider will order a urate-lowering drug for this patient in order to treat the underlying cause of this patient's gout flares. The other drugs manage the flares but do not prevent them from occurring.

13. A patient who takes aspirin for rheumatoid arthritis develops a headache and tinnitus. The patient's lab result includes a plasma salicylate level of 300 µg/mL, and a urine pH of 6.0. What action will the provider take? a. Increase the aspirin dose to better treat the patient's headache. b. Order lab work to identify possible renal toxicity. c. Prepare to provide respiratory support, related to a possible overdose. d. Withhold the aspirin until the patient's symptoms have subsided.

ANS: D This patient shows signs of salicylism, which occurs when ASA levels climb above the therapeutic level. Salicylism is characterized by tinnitus, sweating, headache, and dizziness. Tinnitus is an indication that the maximum acceptable dose has been achieved. Toxicity occurs at a salicylate level of 400 mcg/mL or higher. ASA should be withheld until the symptoms subside and then should be resumed at a lower dose. Increasing the dose would only increase the risk of toxicity. Signs of renal impairment include oliguria and weight gain, which are not present in this patient. This patient has salicylism, not salicylate toxicity, so respiratory support measures are not indicated.

42. A patient who is taking nitrofurantoin reports experiencing several side effects. Which side effect would cause the provider the most concern and would require discontinuation of the medication? a. Anorexia, nausea, and vomiting b. Brown-colored urine c. Drowsiness d. Tingling of the fingers

ANS: D Tingling of the fingers can indicate peripheral neuropathy, which can be an irreversible side effect of nitrofurantoin. The other side effects are not serious and can be reversed.

2. A child will receive 750 mg of an antibiotic for 10 days. The child attends day care. The drug may be dosed in several ways & is available in two concentrations. Which dosing regimen will the nurse discuss with the child's provider? a. 250 mg/5 mL to 375 mg PO twice daily b. 250 mg/5 mL to 250 mg PO three times daily c. 500 mg/5 mL to 250 mg PO three times daily d. 500 mg/5 mL to 375 mg PO twice daily

ANS: D To promote adherence to a drug regimen in children, it is important to consider the size & timing of the dose. In this case the preparation containing 500 mg/5 mL means that a smaller volume can be given, which is more palatable to a child. Twice daily dosing is more convenient for parents, especially when a child is in day care or school; it also helps prevent the problem of the medication being left either at home or at school.

6. A 50-year-old postmenopausal patient who has had a hysterectomy has moderate to severe vasomotor symptoms & is discussing estrogen therapy (ET) with the provider. When the patient expresses concerns about adverse effects of ET, what information will the provider share with her? a. An estrogen-progesterone product will reduce side effects. b. An intravaginal preparation may be best for her. c. Side effects of ET are uncommon among women her age. d. Transdermal preparations have fewer side effects.

ANS: D Transdermal preparations of estrogen have fewer adverse effects, use lower doses of estrogen, & have less fluctuation of estrogen levels than do oral preparations. Progesterone is contraindicated in women who have undergone hysterectomy. Intravaginal preparations are most useful for treating local estrogen deficiency such as vaginal & vulvar atrophy but this patient has severe vasomotor symptoms. Side effects of ET can be common regardless of age.

11. A 60-year-old patient asks about using tretinoin to minimize wrinkles. What response will the provider give? a. This drug is only approved for treating acne. b. Results may be visible within a few weeks of starting therapy. c. Systemic toxicity is a common side effect in patients with sensitive skin. d. The drug is not effective on coarse wrinkles or sun-damaged skin.

ANS: D Tretinoin is used to treat fine wrinkles, not coarse wrinkles, & does not repair sun-damaged skin. It is approved for treatment of both acne & wrinkles. Treatment with tretinoin must continue to maintain the response to the drug. Results are not visible for up to 6 months after beginning therapy. Systemic toxicity is not common.

2. A patient has a localized skin infection, which is most likely caused by a gram-positive cocci. Until the culture and sensitivity results are available, the provider will order a ____-spectrum ____ agent. a. broad; systemic b. broad; topical c. narrow; systemic d. narrow; topical

ANS: D When infections are treated before the causative agent has been identified, and after cultures have been obtained, antibiotics may be used based on the knowledge of which microbes are most likely to cause infection at that particular site. Because this is a localized infection, a topical agent is recommended. Unless the infection is very serious, a narrow-spectrum antibiotic is best.

7. A patient with type 1 diabetes reports taking propranolol for hypertension. What concern does this information present for the provider? a. The β blocker can cause insulin resistance. b. Using propranolol with insulin increases the risk of diabetic ketoacidosis (DKA). c. Propranolol increases insulin requirements because of receptor blocking. d. The β blocker can mask the symptoms of hypoglycemia.

ANS: D β blockers can delay awareness of and response to hypoglycemia by masking signs associated with stimulation of the sympathetic nervous system (e.g., tachycardia, palpitations) that hypoglycemia normally causes. Furthermore, β blockade impairs glycogenolysis, which is one means by which the body can counteract a fall in blood glucose; β blockers, therefore, can worsen insulin-induced hypoglycemia. Propranolol does not cause insulin resistance. The incidence of DKA is not increased by concurrent use of propranolol and insulin. Insulin requirements are not increased because of receptor blocking by propranolol.

11. A child attending daycare is prescribed 750 mg of an antibiotic for 10 days. The drug may be dosed in several ways and is available in two concentrations. Which dosing regimen will the provider consider to best assure drug adherence? a. 375 mg of a 250 mg/5 mL solution PO twice daily b. 250 mg of a 250 mg/5 mL solution PO three times daily c. 250 mg of a 500 mg/5 mL solution PO three times daily d. 375 mg of a 500 mg/5 mL solution PO twice daily

ANS: D d. 375 mg of a 500 mg/5 mL solution PO twice daily To promote adherence to a drug regimen in children, it is important to consider the size & timing of the dose. In this case the preparation containing 500 mg/5 mL means that a smaller volume can be given, which is more palatable to a child. Twice daily dosing is more convenient for parents, especially when a child is in daycare or school; it also helps prevent the problem of the medication being left either at home or at school.

474. What is the most reliable measure for assessing diabetes control over the preceding 3- month period? a. Self-monitoring blood glucose (SMBG) graph report b. Patient's report c. Fasting blood glucose level d. Glycosylated hemoglobin level

ANS: D d. Glycosylated hemoglobin level The glycosylated hemoglobin level tells much about what the plasma glucose concentration has been, on average, over the previous 2 to 3 months. The SMBG graph report is done by the patient & indicates each blood sugar level the patient has on a daily basis. It is not as reliable as the glycosylated hemoglobin level, because the equipment used might not be accurate & the testing may not reflect actual measurements 100% of the time. The patient's report of blood sugar levels is not considered as accurate as the glycosylated hemoglobin level for the same reason that the SMBG is not. One fasting blood glucose level indicates the patient's blood sugar level for that one time when it was obtained.

466. A patient with type 1 diabetes is eating breakfast at 7:30 AM. Blood sugars are on a sliding scale & are ordered before a meal & at bedtime. The patient's blood sugar level is 317 mg/dL. Which formulation of insulin should the nurse prepare to administer? a. No insulin should be administered b. NPH c. 70/30 mix d. Lispro [Humalog]

ANS: D d. Lispro [Humalog] Regular insulin is indicated for sliding scale coverage. Insulin is definitely indicated for this high blood sugar level. NPH is used for scheduled insulin doses & is a longer-acting insulin. A 70/30 mix is also used for scheduled insulin coverage.

470. Which statement is accurate about the long-term complications of diabetes? a. Long-term complications are almost always the result of hypoglycemia & ketoacidosis. b. The complication rates for patients with optimally controlled type 2 diabetes are the same as for those whose disease is not optimally controlled. c. Optimal control of type 1 diabetes produces excessive episodes of life-threatening hypoglycemia. d. Optimal control of both types of diabetes reduces the risk of eye, kidney, & nerve damage.

ANS: D d. Optimal control of both types of diabetes reduces the risk of eye, kidney, & nerve damage. In both types of diabetes, optimal control of the disease slows the development of microvascular complications. Short-term complications are more apt to result from hypoglycemia & ketoacidosis. Patients with type 2 diabetes have fewer complications if their blood sugar level is optimally controlled. Hypoglycemia does not occur more frequently in patients with optimally controlled type 1 diabetes.

12. A 5-year-old has gray teeth. When taking a medication history, the provider will ask about previous use of which group of medications? a. Glucocorticoids b. Salicylates c. Sulfonamides d. Tetracyclines

ANS: D d. Tetracyclines Tetracyclines cause discoloration in developing teeth in children. Glucocorticoids are associated with growth suppression. Salicylates are associated with Reye syndrome. Sulfonamides are associated with kernicterus in newborns.

467. A patient with type 1 diabetes who takes insulin reports taking propranolol for hypertension. Why is the nurse concerned? a. The beta blocker can cause insulin resistance. b. Using the two agents together increases the risk of ketoacidosis. c. Propranolol increases insulin requirements because of receptor blocking. d. The beta blocker can mask the symptoms of hypoglycemia.

ANS: D d. The beta blocker can mask the symptoms of hypoglycemia. Beta blockers can delay awareness of and response to hypoglycemia by masking signs associated with stimulation of the sympathetic nervous system (e.g., tachycardia, palpitations) that hypoglycemia normally causes. Furthermore, beta blockade impairs glycogenolysis, which is one means by which the body can counteract a fall in blood glucose; beta blockers, therefore, can worsen insulin-induced hypoglycemia. Propranolol does not cause insulin resistance. The incidence of DKA is not increased by concurrent use of propranolol and insulin. Insulin requirements are not increased because of receptor blocking by propranolol.

2. A provider orders tamoxifen for a premenopausal woman with ER-positive breast cancer. What rationale supports the provider's decision to prescribe tamoxifen over anastrozole? a. Anastrozole is more likely to cause hot flushes than tamoxifen. b. Anastrozole is more likely to promote endometrial carcinoma. c. Cancer recurrence is higher with anastrozole. d. Anastrozole will not be effective until she is postmenopausal.

ANS: D Anastrozole is used to treat ER-positive breast cancer in postmenopausal women. Because it does not block estrogen production in the ovaries, it is not effective in premenopausal women. It may cause hot flushes but is less likely to do so than tamoxifen. It is devoid of all estrogenic activity and does not promote endometrial cancer.

71. A patient who is taking didanosine reports nausea, vomiting, and abdominal pain. What will the provider recommend to this patient? a. "Take the drug with food to minimize these side effects." b. "Stop taking the drug immediately & resume taking it once your symptoms subside." c. "Take the medication in the evening to avoid experiencing these kinds of symptoms." d. "You will need laboratory tests to determine if these are serious effects of the drug."

ANS: D As with all NRTIs, pancreatitis may occur & may manifest as nausea, vomiting, & abdominal pain. The patient will need evaluation of serum amylase, triglycerides, & calcium. Taking the drug with food or at a different time of day is not indicated. It is not correct to discontinue the drug & to resume it when symptoms subside, since pancreatitis may be fatal.

3. A patient with osteopenia asks about the benefits of hormone therapy in preventing osteoporosis. Which statement by the provider is correct? a. "Estrogen can help reverse bone loss." b. "Hormone therapy increases bone resorption." c. "Hormone therapy does not decrease fracture risk." d. "When hormone therapy is discontinued, bone loss resumes."

ANS: D Benefits of HT to prevent osteoporosis are not permanent; bone loss resumes when HT is discontinued. HT does not reverse bone loss that has already occurred. HT reduces bone resorption. HT can decrease fracture risk.

14. A patient with gout who has increasingly frequent acute gouty attacks will begin receiving allopurinol and colchicine. Which statement will the provider include when teaching the patient about this drug regimen? a. "Allopurinol helps reduce the gastrointestinal side effects of colchicine." b. "Allopurinol reduces the likelihood of acute gouty attacks that often occur when initiating colchicine therapy." c. "The colchicine is given to enhance the effects of the allopurinol." d. "You will take both drugs initially but the colchicine will be discontinued later."

ANS: D Colchicine is used for prophylaxis when urate-lowering drugs, such as allopurinol, are initiated, because gouty episodes have a tendency to increase during this time. Patients start with both drugs, and ultimately the colchicine is withdrawn. Allopurinol does not affect the GI side effects caused by colchicine. Allopurinol may precipitate an acute gouty attack when treatment is begun; colchicine is given to prevent a gouty episode. Colchicine does not enhance the effects of allopurinol.

5. A prescriber provides teaching to a patient about the risks and benefits of estrogen therapy (ET) in the peri-menopausal period. Which statement by the patient indicates understanding of the teaching? a. "Estrogen therapy can provide protection against breast cancer." b. "Estrogen therapy decreases the risk for thrombosis and stroke." c. "Estrogen therapy increases the risk for osteoporosis." d. "Estrogen therapy is the most effective drug for suppression of menopausal symptoms."

ANS: D ET is the most effective means to suppress menopausal symptoms. It can prevent osteoporosis, but it carries risks of breast cancer and stroke.

4. The patient who reports vaginal dryness and pain with intercourse is opposed to using lubricants but is concerned about hormonal adverse effects. Which hormone formulation will the provider choose to decrease systemic estrogen effects? a. Medroxyprogesterone acetate b. Transdermal estrogen c. Low-dose estrogens d. A vaginal conjugated estrogen

ANS: D Estrogens for intravaginal administration are used for local effects, primarily to treat vulval and vaginal atrophy, so there is a lower risk of systemic effects. Medroxyprogesterone is a progesterone used for contraception and is not indicated for this condition. The other formulations will have more systemic effect than the vaginal formulation.

48. A patient is about to begin treatment with isoniazid. When the provider learns that the patient also takes phenytoin for seizures what action will be taken? a. Increasing the phenytoin dose b. Reducing the isoniazid dose c. Monitoring isoniazid levels d. Monitoring phenytoin levels

ANS: D Isoniazid is a strong inhibitor of three cytochrome P450 enzymes, and inhibition of these enzymes can raise the levels of other drugs, including phenytoin. Patients taking phenytoin should have the levels of this drug monitored, and the dose should be reduced if appropriate. Reducing the dose of isoniazid is not indicated. It is not necessary to monitor isoniazid levels.

5. The provider has the patient take a tuberculin skin test and chest x-ray prior to starting etanercept. Testing results are a positive tuberculin skin test with a negative chest radiograph. What action will the provider take? a. Prescribe antituberculosis drugs at the beginning of etanercept therapy. b. Schedule periodic chest radiographs during treatment with etanercept. c. Begin etanercept and advise the patient to report symptoms that indicate conversion from latent to active tuberculosis. d. Prescribe treatment for tuberculosis prior to beginning etanercept treatment.

ANS: D It is important to test all patients for TB. Those who test positive for latent TB should be treated for TB before etanercept treatment is begun. It is not correct to begin TB treatment concurrently with etanercept treatment. Latent TB must be treated and not monitored.

3. A provider prescribing vaccines to a child with an immune deficiency disorder will avoid ordering which vaccination? a. Diphtheria and tetanus toxoids and acellular pertussis (DTaP) vaccine b. Haemophilus influenzae type b (Hib) vaccine c. Polio injection d. Varicella virus vaccine

ANS: D Live vaccines, such as the varicella vaccine, should be avoided by individuals who are immunocompromised. The DTaP vaccine, Hib vaccine, and polio injection may be administered to immunocompromised individuals, because these are not live vaccines.

53. A nurse is preparing to administer oral ofloxacin to a patient. While taking the patient's medication history, the nurse learns that the patient takes warfarin and theophylline. Upon learning this information, what action will the provider take? a. Reduce the dose of ofloxacin. b. Increase the dose of ofloxacin. c. Increase the dose of theophylline. d. Order daily coagulation levels.

ANS: D Ofloxacin increases plasma levels of warfarin, so coagulation tests should be monitored. The ofloxacin dose should not be reduced or increased. Ofloxacin does not affect theophylline levels.

12. A provider teaches a patient who has been diagnosed with hypothyroidism about a new prescription for levothyroxine. Which statement by the patient indicates a need for further teaching? a. "I should not take heartburn medication without consulting my provider first." b. "I should report insomnia, tremors, & an increased heart rate to my provider." c. "If I take a multivitamin with iron, I should take it 4 hours after the levothyroxine." d. "If I take calcium supplements, I may need to decrease my dose of levothyroxine."

ANS: D Patients taking calcium supplements should take these either 4 hours before or after taking levothyroxine, because they interfere with levothyroxine absorption. Many heartburn medications contain calcium, so patients should consult their provider before taking them. Insomnia, tremors, & tachycardia are signs of levothyroxine toxicity & should be reported. Iron also interferes with levothyroxine absorption, so dosing should be 4 hours apart.

5. An 18-month-old child with a 24-hour history of fever & otalgia is seen in the clinic. The child's parents tell the provider that the child was inconsolable during the night. The provider examines the child & notes a bulging, immobile, erythematous tympanic membrane. What action will the provider take to provide the child with effective care? a. Institute watchful waiting & ask the parent to return to the clinic if the child's symptoms worsen. b. Discuss a referral to an ear, nose, & throat specialist for follow-up treatment. c. Advise the parent to give analgesics for 3 days while observing for worsening symptoms. d. Prescribe amoxicillin at 45 mg/kg/dose twice daily.

ANS: D Patients with severe symptoms of AOM should begin treatment with antibiotics upon diagnosis. For children 6 months to 2 years of age, treatment should begin when the diagnosis is certain, as evidenced by a bulging & immobile tympanic membrane (TM) & distinct discomfort. Amoxicillin 45 mg/kg/dose twice daily is indicated. Observation for 2 days is not recommended for this child, because the diagnosis is certain; therefore, asking the parent to return in 2 days or to give only symptomatic treatment is incorrect. Referral to an ear, nose, & throat (ENT) specialist is not recommended unless the child has recurrent AOM or if treatments repeatedly fail.

10. The parent of a child with cerebral palsy reports that the child has pebble-like stools most of the time and seems uncomfortable if several days have passed between stools. Which medication will the provider recommend to the child's parents? a. Bisacodyl suppositories b. Magnesium citrate c. Methylcellulose d. Polyethylene glycol

ANS: D Polyethylene glycol is an osmotic laxative widely used for chronic constipation, which this child has, because it provides relief from abdominal discomfort, improves stool consistency, and increases frequency. Bisacodyl is not recommended for long-term use. Magnesium citrate causes increased water loss, and methylcellulose can also cause impaction if there is insufficient fluid intake.

9. A patient experiences an acute gouty episode despite taking nonsteroidal antiinflammatory drugs for several months. The prescriber and patient had planned to begin therapy with probenecid. What effect will the acute episode have on the patient's planned medication therapy? a. Probenecid will be prescribed as planned, but colchicine will be added to the medication regimen. b. The provider will need to prescribe allopurinol instead of probenecid. c. Probenecid therapy can begin but the patient must commit to increasing fluid intake. d. The probenecid therapy will be delayed until the acute episode has subsided.

ANS: D Probenecid may exacerbate acute episodes of gout, so treatment with this drug should be delayed until the acute attack has passed. Both allopurinol and probenecid can cause or worsen a gout flare. Increasing fluid intake is good advice; however, initiation of probenecid should still be delayed.

11. A patient has been taking psyllium two to three times daily for several days but has not passed a stool. What initial action will the prescriber take when the patient reports abdominal pain? a. Tell the patient to increase water intake. b. Suggest increasing psyllium intake. c. Prescribe a bisacodyl suppository. d. Palpate the patient's abdomen and auscultate for bowel sounds.

ANS: D Psyllium is a bulk-forming laxative and can cause a fecal impaction or obstruction. Abdominal pain can be a sign of impaction or obstruction, so the provider should assess the patient for this complication. Giving fluids by mouth, administering more laxative, or giving a suppository are all contraindicated if an obstruction has developed and should not be done until this condition is ruled out.

36. A young, nonpregnant female patient with a history of a previous urinary tract infection is experiencing dysuria, urinary urgency and frequency, and suprapubic pain of 3 days' duration. She is afebrile. A urine culture is positive for more than 100,000/mL of urine. The provider orders which treatment to assure the most effective treatment for this patient? a. A 14-day course of amoxicillin with clavulanic acid b. A 7-day course of ciprofloxacin c. A single dose of fosfomycin d. A 3-day course of trimethoprim/sulfamethoxazole

ANS: D Short-course therapy is recommended for uncomplicated, community-acquired lower urinary tract infections. The short course is more effective than a single dose, and compared with longer course therapies, it is less costly, has fewer side effects, and is more likely to foster compliance. Amoxicillin with clavulanic acid is a second-line drug used for pyelonephritis. Fosfomycin is a second-line drug and can be useful in patients with drug allergies.

7. An adult patient anemia following cancer chemotherapy will now begin treatment with testosterone. Which statement by the patient indicates understanding of the provider's teaching? a. "Facial hair may develop with this drug but will go away over time." b. "I may experience an increase in breast size while taking this drug." c. "Testosterone may increase my high-density lipoprotein (HDL) cholesterol and reduce my low-density lipoprotein (LDL) cholesterol." d. "Testosterone treats anemia by stimulating the synthesis of a renal hormone."

ANS: D Testosterone can be used to treat refractory anemias in men and women. It works by stimulating the synthesis of erythropoietin, a renal hormone that stimulates the production of red blood cells. Virilization effects can be permanent if the hormone is not withdrawn, so patients developing facial hair and other signs should be told to report this to the provider. Breast enlargement occurs in males taking this drug. Testosterone reduces HDL cholesterol and increases LDL cholesterol.

2. A provider is caring for a patient who has undergone organ transplantation. Because the major histocompatibility complex (MHC) molecules of the donor are different from those of the patient, the provider will prescribe a drug from which drug class? a. Antibiotics b. Antihistamines c. Immune globulins d. Immunosuppressants

ANS: D The MHC molecules from one individual are recognized as foreign by the immune system of another individual; therefore, when an attempt is made to transplant organs between individuals who are not identical twins, immune rejection of the transplant is likely. Immunosuppressants are given to counter this response. Antibiotics are used to destroy bacteria. Antihistamines block hypersensitivity reactions. Immune globulins are given to confer passive immunity when specific acquired immunity has not yet developed a response.

25. A provider has ordered intravenous gentamicin at a dose that is half the usual dose for an adult patient. What factor noted in the patient's medical history would be a likely reason for this action? a. Antibiotic resistance b. Interpatient variation c. Liver disease d. Renal disease

ANS: D The aminoglycosides are eliminated primarily by the kidneys, so in patients with renal disease, doses should be reduced or the dosing interval should be increased to prevent toxicity. Patients with antibiotic resistance would be given amikacin. Interpatient variation may occur but cannot be known without knowing current drug levels. Aminoglycosides are not metabolized by the liver, so liver disease would not affect drug levels.

3. A prescriber is providing education to a patient who is beginning therapy with testosterone gel. Which statement made by the patient demonstrates a need for further teaching? a. "This drug puts me at an increased risk for blood clots." b. "I will need to tell my pregnant wife not to handle my laundry." c. "I should have my blood drawn for laboratory tests in 14 days." d. "I should apply the medication to my genitals for best results."

ANS: D The medication should be applied to the arms, shoulders, and abdomen, not to the genitalia; this statement indicates a need for further teaching. Secondary exposure to testosterone gel can occur with handling unwashed clothing. The patient should be advised to return to the clinic within 2 weeks for blood tests.

70. A patient starting therapy with efavirenz asks about the timing of the medication with regard to meals. What patient education about the administration of this medication should the prescriber provide? a. The drug must be taken within 30 minutes after a meal. b. The drug is best taken with a high-fat meal. c. The drug can be taken anytime without regard to meals. d. The drug should be taken once daily on an empty stomach.

ANS: D The nurse should advise the patient that the medication should be taken once daily on an empty stomach. Thirty minutes after a meal is too soon to take the medication. The medication is taken on an empty stomach, because high-fat meals increase plasma levels by 39% with capsules & by 79% with tablets. The medication must not be taken with high-fat meals.

40. Which patient diagnosed with a urinary tract infection will be hospitalization and prescribed intravenous antibiotics? a. A 5-year-old child with a fever of 100.5°F, dysuria, and bacteriuria b. A pregnant woman with bacteriuria, suprapubic pain, and fever c. A young man with dysuria, flank pain, and a previous urinary tract infection d. An older adult man with a low-grade fever, flank pain, and an indwelling catheter

ANS: D The patient with an indwelling catheter and signs of pyelonephritis shows signs of a complicated UTI, which is best treated with intravenous antibiotics. Three other patients show signs of uncomplicated urinary tract infections that are not severe and can be treated with oral antibiotics.

15. An older male patient takes furosemide and low-dose aspirin. Urine output is low despite the diuretic. Today's blood pressure is 140/80 mm Hg and the serum creatinine and blood urea nitrogen (BUN) levels are elevated. The patient has also gained 10-pounds over the past 3 months. What action will the provider discuss with the patient? a. Adding an antihypertensive medication b. Recheck the serum creatinine and BUN c. Ordering a potassium-sparing diuretic d. Withdrawing the aspirin

ANS: D This patient shows signs of renal impairment, as evidenced by weight gain despite the use of diuretics, decreased urine output, hypertension, and elevated serum creatinine and BUN. Aspirin can cause acute, reversible renal impairment and should be withdrawn. Hypertensive medications do not treat the underlying cause. Rechecking the serum creatinine and BUN are not indicated because elevated values are typical for the situation presented. Addition of a potassium-sparing diuretic is not indicated.

22. A patient plans to perform missionary work in a region with poor drinking water. The provider gives the patient a prescription for ciprofloxacin to take on the trip. What instructions will the prescriber discuss with this patient? a. Combine the antibiotic with an antidiarrheal medication, such as loperamide. b. Start the ciprofloxacin 1 week before traveling. c. Take 1 tablet of ciprofloxacin with each meal while living in areas with inadequate water maintenance. d. Use the drug if symptoms develop and are severe or do not improve in a few days.

ANS: D Traveler's diarrhea is generally caused by Escherichia coli; treatment is usually unnecessary, because the disease runs its course in a few days. If symptoms are severe or prolonged, an antibiotic, such as ciprofloxacin, may be helpful. Patients should be instructed to take it only if needed. Antidiarrheal medications may just slow the export of the organism and prolong the course of the disease, but they may be used when symptoms are mild for relief from discomfort. Prophylactic treatment with antibiotics is not recommended. Ciprofloxacin is given twice daily, not with meals.

10. An adolescent patient with moderate acne has begun a regimen consisting of combination clindamycin/benzoyl peroxide & tretinoin. Which statement by the patient indicates understanding of this medication regimen? a. "I should apply the Retin-A immediately after bathing." b. "I should apply the Retin-A twice daily." c. "I should augment this therapy with an abrasive soap." d. "I should use sunscreen every day."

ANS: D Tretinoin increases susceptibility to sunburn, so patients should be warned to apply a sunscreen & wear protective clothing. Before applying Retin-A, the skin should be washed, toweled dry, & allowed to dry fully for 15 to 30 minutes. Retin-A is applied once daily. Abrasive soaps intensify localized reactions to Retin-A & should not be used.

16. A patient develops Clostridium difficile-associated diarrhea (CDAD). Which antibiotic will the prescriber order to treat this infection? a. Chloramphenicol b. Clindamycin c. Linezolid d. Vancomycin

ANS: D Vancomycin and metronidazole are the drugs of choice for treating CDAD.

59. A provider has ordered oral voriconazole for a patient who has a systemic fungal infection. The nurse obtains a medication history and learns that the patient takes phenobarbital for seizures. When the nurse contacts the provider what action will the provider take? a. Confirm the intravenous voriconazole order. b. Reduce the dose of phenobarbital. c. Reduce the dose of voriconazole. d. Prescribe a different antifungal agent.

ANS: D Voriconazole can interact with many drugs. It should not be combined with drugs that are powerful P450 inhibitors, including phenobarbital, because these can reduce the levels of voriconazole. Administering the voriconazole IV will not increase the serum level. It is not correct to reduce the dose of either drug.

60. A patient has an invasive aspergillosis infection. Which antifungal agent will the prescriber consider the drug of choice for this infection? a. Amphotericin B b. Fluconazole c. Posaconazole d. Voriconazole

ANS: D Voriconazole has replaced amphotericin B as the drug of choice for treating invasive aspergillosis. Fluconazole, which is fungistatic, is not used to treat aspergillosis. Posaconazole is used for prophylaxis of aspergillosis in immunocompromised patients.

468. A nurse provides dietary counseling for a patient newly diagnosed with type 1 diabetes. Which instruction should be included? a. "You may eat any foods you want & cover the glucose increase with sliding scale, regular insulin." b. "Most of the calories you eat should be in the form of protein to promote fat breakdown and preserve muscle mass." c. "Your total caloric intake should not exceed 1800 calories in a 24-hour period." d. "You should use a carbohydrate counting approach to maintain glycemic control."

ANS: D d. "You should use a carbohydrate counting approach to maintain glycemic control." Patients with diabetes should be given intensive insulin therapy education using either a carbohydrate counting or experience-based estimation approach in achieving glycemic control. A patient with diabetes cannot eat any foods desired & then cover the glucose increase with a sliding scale of regular insulin. Evidence suggests that there is not an ideal percentage of calories that should be ingested from carbohydrate, fat, or protein. Every patient with diabetes must be assessed individually to determine the number of total calories the person should have daily. The total caloric intake should be spread evenly throughout the day, with meals spaced 4 to 5 hours apart.


Kaugnay na mga set ng pag-aaral

AICE Business Unit 3 Multiple Choice

View Set

152 TEST 2 (Culture & Family Dynamics)

View Set

Chapter 15 Caring for the Postpartal woman

View Set

U.S. History Semester 1 Final Exam Review

View Set